<<

43 Maurizio Fava, MD, and George I. Papakostas, MD

KEY POINTS heterogeneous group of compounds that have been tradition- ally subdivided into major groups according to their chemical • The immediate mechanism of action of modern structure or, more commonly, according to their effects on antidepressants (“immediate effects”) involves monoamine neurotransmitter systems: selective influencing the function of one or more monoamine re-uptake inhibitors (SSRIs); TCAs and the related cyclic neurotransmitter systems (serotonin, antidepressants (i.e., and ); MAOIs; [noradrenaline], or ). serotonin norepinephrine re-uptake inhibitors (SNRIs); nore- pinephrine re-uptake inhibitors (NRIs); norepinephrine/ • Influencing function has been shown dopamine re-uptake inhibitors (NDRIs); serotonin receptor to result in several changes in second-messenger antagonists/; and alpha2- receptor antago- systems and gene expression/regulation (“downstream nists. Because they overlap, the mechanisms of action and effects”). the indications for use for the antidepressants are discussed • “Downstream effects” may explain the delayed onset together, but separate sections are provided for their method of of response seen with all administration and their side effects. contemporary agents (most patients improve following at least 3 weeks of treatment). MECHANISM OF ACTION • For the most part, all contemporary antidepressants are equally effective when treating major depressive The precise mechanisms by which the antidepressant disorder. exert their therapeutic effects remain unknown, although much is known about their immediate actions within the • There are significant differences in the relative nervous system. All of the currently marketed antidepressants tolerability and safety of contemporary interact with the monoamine neurotransmitter systems in the antidepressants. brain, particularly the norepinephrine and serotonin systems, and to a lesser extent the dopamine system. Essentially all currently marketed antidepressants have as their molecular targets components of monoamine synapses, including the OVERVIEW re-uptake transporters (that terminate the action of norepine- phrine, serotonin, or dopamine in synapses), monoamine A large number of compounds have been developed to treat receptors, or that serve to metabolize monoamines. depression. Traditionally, these compounds have been called What remains unknown is how these initial interactions “antidepressants,” even though most of these drugs are also produce a therapeutic response.5 The search for the molecular effective in the treatment of a number of anxiety disorders (such events that convert altered monoamine neurotransmitter func- as panic and obsessive-compulsive disorder [OCD]) and a tion into the lifting of depressive symptoms remains a matter variety of other conditions (Box 43-1). The precursors of two of of very active research. the major contemporary antidepressant families, the monoam- Since TCAs and MAOIs were the first antidepressants to be ine oxidase inhibitors (MAOIs) and the antidepres- discovered and introduced, this was initially interpreted as sug- sants (TCAs), were discovered by serendipity in the 1950s. gesting that antidepressants work by increasing noradrenergic Specifically, the administration of , an anti- or neurotransmission, thus compensating for a mycobacterial agent, was first noted to possess antidepressant postulated state of relative monoamine “deficiency.” However, effects in depressed patients suffering from tuberculosis.1 this simple theory could not fully explain the action of antide- Shortly thereafter, iproniazid was found to inhibit MAO, pressant drugs for a number of reasons. The most important of which is involved in the catabolism of serotonin, norepine- these include the lack of convincing evidence that depression phrine (noradrenaline), and dopamine. is characterized by a state of inadequate or “deficient” mono­ In parallel, was initially developed as an anti- neurotransmission. In fact, the results of studies testing histamine, but Kuhn2 discovered that of some 500 imipramine- the monoamine depletion hypothesis in depression have treated patients with various psychiatric disorders, only those yielded inconsistent results.5 Moreover, blockade of mono­ with endogenous depression with mental and motor retarda- amine re-uptake or inhibition of monoamine degradation tion showed a remarkable improvement during 1 to 6 weeks of occurs rapidly (within hours) following monoamine re-uptake daily imipramine therapy. The same compound was also found inhibitor or MAOI administration, respectively. However, to inhibit the re-uptake of serotonin and norepinephrine.3,4 treatment with antidepressants for less than 2 weeks is unlikely Thus, it was the discovery of the antidepressant effects of ipro- to result in a significant lifting of depression; it has been con- niazid and imipramine that led to the development of the sistently observed and reported that remission of depression MAOIs and TCAs, and this discovery was instrumental in the often requires 4 weeks of treatment or more. These considera- formulation of the monoamine theory of depression. In turn, tions have led to the idea that inhibition of monoamine guidedy b this theory, the subsequent development of com- re-uptake or inhibition of MAO by antidepressants represents pounds selective for the re-uptake of either serotonin or nore- an initiating event. The actual therapeutic actions of antide- pinephrine or both was designed, rather than accidental. As a pressants, however, result from slower adaptive responses result, over the last few decades, chemical alterations of these within neurons to these initial biochemical perturbations first antidepressants have resulted in the creation of a wide (“downstream events”).6 Although research geared toward variety of monoamine-based antidepressants with a variety understanding the therapeutic actions of antidepressants of mechanisms of action. The antidepressant drugs are a has been challenging, receptor studies have been useful in

489

Downloaded for Rohul Amin ([email protected]) at Uniformed Services Univ of the Health Sciences from ClinicalKey.com by Elsevier on September 21, 2018. For personal use only. No other uses without permission. Copyright ©2018. Elsevier Inc. All rights reserved. 490 PART X Treatment Approaches

by catechol O-methyltransferase (COMT), an that acts BOX 43-1 Possible Indications for Antidepressants extracellularly. The classification of antidepressant drugs has perhaps • Major depressive disorder and other unipolar depressive focused too narrowly on synaptic (i.e., “imme- disorders diate effects”), and has certainly failed to take into account • Bipolar depression molecular and cellular changes in neural function that are • Panic disorder brought about by the chronic administration of these agents.5 • Social anxiety disorder For example, it has been postulated that changes in post- • Generalized anxiety disorder receptor signal transduction may account for the aforemen- • Post-traumatic stress disorder tioned characteristic lag-time between the time a is • Obsessive-compulsive disorder (e.g., and SSRIs) administered and the drug-induced resolution of a depressive • Depression with psychotic features (in combination with an episode. In fact, studies have shown that hippocampal neuro- drug) genesis occurs following chronic antidepressant treatment • Bulimia nervosa in animal models.8 In parallel, rapid activation of the TrkB • Neuropathic pain (tricyclic drugs and SNRIs) neurotrophin receptor and PLC gamma-1 signaling has • (e.g., , ) been described with almost all antidepressant drugs, a possible • Enuresis (imipramine best studied) mechanism by which the process of neuronal neurogenesis • Atypical depression (e.g., monoamine oxidase inhibitors) observed following chronic administration of antidepressants • Attention-deficit/hyperactivity disorder (e.g., , may. occur 9 Alternatively, one could postulate that this ) lag phase in antidepressant action may be related to a SNRIs, Serotonin norepinephrine re-uptake inhibitors; SSRIs, selective re-organization of neuronal networks, postulated as a potential 5 serotonin re-uptake inhibitors. “final common pathway” for antidepressant effects to occur. Nevertheless, further research is urgently needed in order to help us understand the specific effects of the antidepressants and what constitutes illness and recovery in depression. understanding and predicting some of the side effects of con- temporary antidepressants. For example, the binding affinity of antidepressants at muscarinic receptors generally Mechanism of Action of Selective Serotonin parallels the prevalence of certain side effects during treatment Re-uptake Inhibitors (e.g., dry mouth, , urinary hesitancy, poor concen- tration). Similarly, treatment with agents that have high affini- At therapeutically relevant doses, the SSRIs exhibit significant effects primarily on serotonin re-uptake in the human brain.10 ties for histamine H1 receptors (e.g., and amitriptyline) appears to be more likely to result in sedation, and increased Some SSRIs also appear to have effects on other monoamine appetite. Such information is very useful to clinicians and transporters, with demonstrating modest dopamine re-uptake inhibition, and and demon- patients when making treatment decisions or to researchers 10 when attempting to develop new antidepressants. strating modest norepinephrine re-uptake inhibition. In The architecture of the monoamine neurotransmitter addition, fluoxetine, particularly the R-isomer, has mild 5-HT2A systems in the central nervous system (CNS) is based on the and 5-HT2C antagonist activity. Non-monoaminergic effects synthesis of the neurotransmitter within a restricted number have also been described for some of the SSRIs, including moderate and selective effects on expres- of nuclei within the brainstem, with neurons projecting widely 11 throughout the brain and, for norepinephrine and serotonin, sion and editing. The SSRIs have minimal or no affinity the spinal cord as well.5 Norepinephrine is synthesized within for muscarinic cholinergic, histaminergic, and adrenergic a series of nuclei in the medulla and pons, of which the largest receptors, with the exception of paroxetine (which is a weak is the nucleus locus coeruleus. Serotonin is synthesized in the cholinergic ), (which is a weak antagonist of the histamine H1 receptor), and sertraline (which brainstem raphe nuclei. Dopamine is synthesized in the sub- 10 stantia nigra and the ventral tegmental area of the midbrain. has weak affinity for the alpha1 receptors). Overall, the affin- Through extensive projection networks, these neurotransmit- ity of these agents for these specific receptors is lower than ters influence a large number of target neurons in the cerebral those of the TCAs, resulting in a milder side-effect profile. cortex, basal forebrain, striatum, limbic system, and brain- Similarly, the lack of significant action for the remaining SSRIs stem, where they interact with multiple receptor types to regu- on these receptors is also thought to contribute to the milder late arousal, vigilance, attention, sensory processing, emotion, side-effect profile of these agents compared with the TCAs. and cognition (including memory).7 Norepinephrine, serotonin, and dopamine are removed Mechanism of Action of Serotonin Norepinephrine from synapses after release by re-uptake, mostly into pre- Re-uptake Inhibitors synaptic neurons.5 This mechanism of terminating neurotrans- mitter action is mediated by specific norepinephrine, serotonin, Unlike the TCAs, SNRIs inhibit the re-uptake of serotonin and dopamine re-uptake transporter proteins. After re-uptake, more potently than the re-uptake of norepinephrine.10 Similar norepinephrine, serotonin, and dopamine are either re-loaded to most SSRIs, the SNRIs have minimal or no affinity for into vesicles for subsequent release or broken down by MAO. muscarinic cholinergic, histaminergic, and adrenergic recep- 10 MAO is present in two forms (MAOA Oand MA B), which differ tors. Interestingly enough, administration of these drugs has in their substrate preferences, inhibitor specificities, tissue been shown to prevent a decrease in cell proliferation and expression, and cell distribution. MAOA preferentially oxidizes BDNF expression in rat hippocampus observed with chronic serotonin and is irreversibly inactivated by low concentrations stress, in a study that offers further insights into the “down- 12 of the acetylenic inhibitor clorgyline. MAOB preferentially oxi- stream effects” of these agents. In parallel, studies suggest dizes phenylethylamine (PEA) and benzylamine and is irre- that chronic treatment with the SNRI not only versibly inactivated by low concentrations of pargyline and produces a marked up-regulation of BDNF mRNA and protein deprenyl.5 ,Dopamine , and are substrates buty ma also affect the sub-cellular re-distribution of neuro- for both forms of MAO. are also broken down trophin, potentially improving synaptic plasticity.13

Downloaded for Rohul Amin ([email protected]) at Uniformed Services Univ of the Health Sciences from ClinicalKey.com by Elsevier on September 21, 2018. For personal use only. No other uses without permission. Copyright ©2018. Elsevier Inc. All rights reserved. Antidepressants 491

Mechanism of Action of Norepinephrine appears that the net effect of at release-regulating 5-HT1A autoreceptors is inhibitory, leading to markedly 43 Re-uptake Inhibitors increased serotonin output. At therapeutically relevant doses, the NRIs have significant is a recently-approved antidepressant with effects primarily on norepinephrine re-uptake, although the multi-modal activity that functions as a serotonin 5-HT3, 5-HT7 14 NRI is also a weak inhibitor of serotonin uptake. and 5-HT1D receptor antagonist, serotonin 5-HT1B receptor NRIs also appear to have several non-monoaminergic proper- partial , serotonin 5-HT1A receptor agonist and inhibi- ties. Specifically, the NRI also appears to function- tor of the in vitro.37 Vortioxetine has ally inhibit nicotinic receptors.15 In addition, in shown to be able to increase extracellular serotonin, dopamine, rats, atomoxetine has been shown to increase in vivo extracel- and norepinephrine in the medial prefrontal cortex and ventral lular levels of acetylcholine (ACh) in cortical but not sub- hippocampus,38 and to significantly increase cell proliferation cortical brain regions, with a mechanism dependent on and cell survival and stimulate maturation of immature norepinephrine alpha1 and/or dopamine D1 receptor activa- granule cells in the sub-granular zone of the dentate gyrus of tion.16 Furthermore, the major human metabolite of atomox- the hippocampus after 21 days of treatment.37 etine (4-hydroxyatomoxetine) is a of the kappa- receptor.17 , Finally reboxetine has also been found to be the antidepressant that affects glutamate receptors Mechanism of Action of Norepinephrine/ (GluR) most, with a decrease of GluR3 expression.11 Dopamine Re-uptake Inhibitors The NDRIs primarily block the re-uptake of dopamine and norepinephrine and have minimal or no affinity for pre- Mechanism of Action of Serotonin Receptor synaptic or post-synaptic monoamine receptors. The mecha- Agonist/Antagonists nism of action of bupropion has not been fully elucidated, although it appears to primarily block the re-uptake of both Both trazodone and are relatively weak inhibitors dopamine and norepinephrine.39 Bupropion and its metabo- of serotonin and norepinephrine uptake, and they primarily lites have been shown to be able to inhibit striatal uptake of block serotonin 5-HT receptors (in some cases, demonstrat- 2A thee selectiv (DAT)-binding radioligand ing partial agonist properties as well).18–21 They also share a (11)C-betaCIT-FE in vivo,40 and to have mild affinity for the metabolite, m-chlorophenylpiperazine (mCPP), which acts as norepinephrine transporter,41 although some researchers have a serotonin 5-HT agonist and appears to be able to release argued that the effect of bupropion on norepinephrine is pri- serotonin pre-synaptically.22 Trazodone also appears to stimu- marily through an increase in pre-synaptic norepinephrine late the mu - and mu -opioid receptors23 and is a potent 1 2 release.42 Whatever the exact mechanism may be, it appears agonist of the serotonin 5-HT ,receptors which, when acti- 2C that the overall effect of bupropion is a dose-dependent vated,24,25 yma inhibit NMDA-induced cyclic GMP elevation. increase in brain extracellular dopamine and norepinephrine Since trazodone is also a weak inhibitor of serotonin re-uptake concentrations.43 It also appears that bupropion acts as an as well, the overall effect of trazodone appears to be an increase antagonist at alpha beta and alpha beta nAChRs in rat stria- in extracellular levels of serotonin in the brain.26 This effect 3 2 3 4 tum and hippocampus, respectively, across the same concen- explains the fact that trazodone treatment has been associated tration range that inhibits DAT and norepinephrine transporter with the occurrence of a serotonin syndrome.27 Both trazo- (NET) function.44 done and (although to a lesser degree) nefazodone are potent blockers of the alpha1-. , available in Europe as an antidepressant, is a Mechanism of Action of Alpha2-Adrenergic selective 5-HT2C antagonist and acts as a melatonin MT1 and Receptor Antagonists MT2 receptor agonist. The 5-HT2C antagonism properties of agomelatine have been thought to be responsible for increases The alpha2-adrenergic receptor antagonists (e.g., in frontocortical dopaminergic and adrenergic activity in and ) appear to enhance the release of both serot- 28 animals during administration of agomelatine. onin and norepinephrine by blocking auto- and hetero-alpha2 and act as full agonists at serotonin receptors.45 Since mirtazapine appears to be a blocker of sero- 5-HT1A autoreceptors and are generally, but not exclusively, tonin 5-HT2 and 5-HT3 receptors as well, it is thought to 29 partial agonists at post-synaptic serotonin 5-HT1A .receptors enhance the release of norepinephrine and also enhance 46 Neither buspirone or gepirone are approved for depression, 5-HT1A–mediated serotonergic transmission. Mirtazapine but buspirone is FDA-approved for the treatment of anxiety. was the first alpha2-adrenergic receptor antagonist to be Buspirone and gepirone show weak alpha1-adrenoceptor affin- approved by the Food and Drug Administration (FDA) for ity, but significant and selective alpha1-adrenoceptor intrinsic depression. Mirtazapine is also a potent histaminergic H1- 10 , which was expressed in a tissue- and species-dependent receptor antagonist. Mianserin, also an alpha2-noradrenergic 30 manner. Buspirone also shows binding with high affinity to receptor antagonist and a serotonin 5-HT2 antagonist, is avail- recombinant human D3 and D4 receptors (~98 and ~29 nm able in Europe and is not FDA-approved. respectively).31 Buspirone and gepirone are thought to lead 32 to excitation of noradrenergic cell firing, antagonizing Mechanism of Action of Tricyclic Antidepressants primarily pre-synaptic inhibitory dopamine D2 autoreceptors at dopaminergic neurons.33 Buspirone also has potent TCAs, referred to as such because they share a chemical struc- alpha2-adrenoceptor antagonist properties via its principal ture with two joined rings, have been in use for metabolite, 1-(2-pyrimidinyl)-.34,35 Vilazodone is a almost half a century for the treatment of depression. With serotonin 5-HT1A receptor partial agonist and a selective sero- the exception of clomipramine, and in contrast with the tonin re-uptake inhibitor that induces maximal serotonin SNRIs, the TCAs inhibit the re-uptake of norepinephrine more levels that are similar in the medial and the lateral cortex and potently than the re-uptake of serotonin. Recent studies have are up to six-fold higher than those induced by SSRIs tested also shown that five of the TCAs bind to the S1S2 domain of in parallel, when serotonin levels are measured in two the GluR2 subunit of the AMPA receptor, suggesting an effect sub-regions of the rat prefrontal cortex by microdialysis.36 It of TCAs on the glutamergic system.47 In addition, doxepin,

Downloaded for Rohul Amin ([email protected]) at Uniformed Services Univ of the Health Sciences from ClinicalKey.com by Elsevier on September 21, 2018. For personal use only. No other uses without permission. Copyright ©2018. Elsevier Inc. All rights reserved. 492 PART X Treatment Approaches amitriptyline, and also inhibit uptake by More recently, additional pharmacological properties for blocking the glycine transporter 1b (GLYT1b) and glycine trans- the MAOIs have been revealed. MAOIs, for instance, also 48 porter 2a (GLYT2a) to a similar extent. Amoxapine displays a appear to inhibit the binding of [3H] quinpirole, a dopamine selective inhibition of GLYT2a behaving as a 10-fold more effi- agonist with high affinity for D2 and D3 dopamine recep- 48 58,59 cient inhibitor of this isoform than of GLYT1b and is also a tors. oT complicate the pharmacology of MAOIs further, 49 dopamine D2 receptor antagonist. Interestingly, in vitro data two of the MAOIs, and , have meth- suggest that and clomipramine have compara- and amphetamine as metabolites.60,61 In 50 ble affinity for the dopamine D2 . receptor The TCAs, to addition, also elevates brain gamma-aminobutyric varying degrees, are also fairly potent blockers of histamine acidA) (GAB levels, and as yet unidentified metabolites of 61 H1 ,receptors serotonin 5-HT2 ,receptors muscarinic acetylcho- phenelzine may be responsible for this effect. R(−)- but not 10,50 line receptors, and alpha1-adrenergic receptors. S(+)- selegiline also appears to induce dopamine release by directly modulating ATP-sensitive potassium channels.62 Mechanism of Action of Monoamine Finally, (+)-tranylcypromine (TCP) has been shown to be Oxidase Inhibitors more potent than (−)-TCP as an inhibitor of 5-HT uptake, whereas (−)-TCP has been shown to be more potent than MAOIs act by inhibiting MAO, an enzyme found on the outer (+)-TCP as an inhibitor of dopamine and norepinephrine membrane of mitochondria, where it catabolizes (degrades) a uptake.63 number of monoamines including dopamine, norepine- Although the risk for serotonin syndromes may be lower phrine, and serotonin. Specifically, following their re-uptake than with the older MAOIs, and a number of studies suggested from the synapse into the pre-synaptic neuron, norepine- the safety of combining with SSRIs,64–66 there phrine, serotonin, and dopamine are either loaded into vesi- have been a number of non-fatal67,68 and fatal serotonin syn- cles for subsequent re-release or broken down by MAO. MAO dromes involving the co-administration of moclobemide 69–74 is present in two forms (MAOA Oand MA B), which differ in and. SSRIs For these reasons, the concomitant use of their substrate preferences, inhibitor specificities, tissue expres- moclobemide and serotonergic agents should be avoided. In sion, and cell distribution. MAOA preferentially oxidizes sero- addition, the co-ingestion of moclobemide and SSRIs in over- tonin and is irreversibly inactivated by low concentrations of dosey ma result in death, which needs to be taken into account 70 the acetylenic inhibitor clorgyline. MAOB preferentially oxi- for patients at risk for suicide. dizes phenylethylamine (PEA) and benzylamine and is irre- versibly inactivated by low concentrations of pargyline and CLINICAL USES OF ANTIDEPRESSANTS deprenyl. Dopamine and the dietary (exogenous) , tyramine and tryptamine, are substrates for both forms of Since the introduction of fluoxetine, the SSRIs and the SNRIs MAO.51 In the gastrointestinal (GI) tract and the , MAO have become the most often prescribed initial pharmacologi- catabolizes a number of dietary pressor amines (such as cal treatment for major depressive disorder (MDD). The dopamine, tyramine, tryptamine, and phenylethylamine).52 success of these newer agents in displacing TCAs as first-choice For this reason, consumption of certain foods (that contain agents is not based on established differences in efficacy, but high levels of dietary amines) while on an MAOI may rather on a generally more favorable side-effect profile (such precipitate a hypertensive crisis, characterized by hyperten- as a lack of and cardiac side effects, and a high sion, hyperpyrexia, , tremulousness, and cardiac therapeutic index, i.e., the ratio of lethal dose to therapeutic arrhythmias.53 The same reaction may also occur during dose), combined with ease of administration. Furthermore, co-administration of dopaminergic agents and MAOIs, while with certain co-morbidities of depression (such as OCD), the co-administration of MAOIs with other antidepressants SSRIs offer advantages in efficacy over the TCAs. Nonetheless, that potentiate serotonin could result in serotonin syndromes the TCAs remain useful alternatives for the treatment of some due to toxic CNS serotonin levels. The serotonin syndrome is patients with depression. In contrast, because of their inferior characterized by alterations in cognition (e.g., disorientation safety profile, the traditional MAOIs are a class of drugs and confusion), behavior (e.g., agitation and restlessness), reserved for patients in whom other treatments have failed. autonomic nervous system function (e.g., fever, shivering, dia- Clearly, the newer antidepressants (SSRIs, SNRIs, NRIs, NDRIs, phoresis, and diarrhea), and neuromuscular activity (e.g., and serotonin receptor antagonists) all have major safety or ataxia, hyperreflexia, and myoclonus).54–56 OSince MA enzy- tolerability advantages over the TCAs and MAOIs. The recently- matic activity requires approximately 14 days to be fully approved serotonin receptor antagonist vortioxetine may have restored, such food or should be avoided for 2 the additional advantage of distinctive pro-cognitive effects. weeks following the discontinuation of an irreversible MAOI (“MAOI washout period”). Serotonergic and dopaminergic Continuation and Maintenance of antidepressants are typically discontinued 2 weeks before the Antidepressant Treatment initiation of an MAOI, with the exception of fluoxetine, which needs to be discontinued 5 weeks in advance because of its Originally, based on studies with TCAs, patients with unipolar much longer half-life. depressive disorders were observed to be at high risk for Older MAOIs, including phenelzine, tranylcypromine, and relapse when treatment was discontinued within the first 16 , irreversibly inhibit the enzymatic activity of weeks of therapy. Therefore, in treatment-responders, most bothO MA A Oand MA B, while newer ones are relatively selec- experts favor a continuation of antidepressant therapy for a tive ( and moclobemide preferentially inhibit minimum of 6 months following the achievement of remis- MAOA; oral selegiline selectively inhibits MAOB). Reversible sion. The value of continuation therapy for several months to MAOA-selective inhibitors are designed to minimize the risk prevent relapse into the original episode has also been estab- of hypertensive crises, and patients on conventional doses lished for virtually all of the newer agents.75 Risk of recurrence of moclobemide do not need to strictly adhere to the low- after this 6- to 8-month continuation period (i.e., the devel- tyramine diet, although, at very high doses (e.g., 900 mg/day opment of a new episode after recovery from the index 57 of moclobemide), inhibition of MAOB also occurs. All episode) is particularly elevated in patients with a chronic MAOIs available in the US are irreversible inhibitors of MAOA course before recovery, residual symptoms, and multiple 76 andO MA B .activity prior episodes (three or more). For these individuals, the

Downloaded for Rohul Amin ([email protected]) at Uniformed Services Univ of the Health Sciences from ClinicalKey.com by Elsevier on September 21, 2018. For personal use only. No other uses without permission. Copyright ©2018. Elsevier Inc. All rights reserved. Antidepressants 493 optimal duration of maintenance treatment is unknown, but tion of adult and pediatric patients treated with these drugs for is assumed to be much longer (measured in years). In fact, worsening depression or the emergence of suicidality. This 43 based on research to date, prophylactic efficacy of an antide- warning was based on the analyses of clinical trials data that pressant has been observed for as long as 5 years with clear compared the relative risk of emergence of suicidal ideation on benefit.77 In contrast to the initial expectation that mainte- these drugs compared to following initiation of treat- nance therapy would be effective at dosages lower than that ment. The difference was small, but statistically significant. required for acute treatment, the current consensus is that This finding underscores the need for good practice, which full-dose therapy is required for effective prophylaxis.78 About includes education of patients (and families if the patient is a 20% to 30% of patients who are treated with each of the child) about side effects of drugs (including the possible emer- classes of antidepressants will experience a return of depres- gence of suicidal thoughts and behaviors), close monitoring sive symptoms despite continued treatment. In such patients, (especially early in treatment), and the availability of a clini- a dose increase of the antidepressant is typically the first-line cian in case suicidality emerges or worsens. A general consen- approach.79 sus, remains however, that the risks associated with withholding antidepressant treatment from patients, including pediatric Suicide Risk patients, with serious depression vastly outweighs the risks associated with the drugs by many orders of magnitude. Unlike the SSRIs and other newer agents, the MAOIs, TCAs, and related cyclic antidepressants (maprotiline and amoxap- CHOICE OF AN ANTIDEPRESSANT ine) are potentially lethal in overdose. Thus, a careful evalua- tion of impulsiveness and suicide risk influences not only the A large number of antidepressants are available (Table 43-1), decision as to the need for hospitalizing a person with depres- including SSRIs, SNRIs, NRIs, NDRIs, serotonin receptor sion but also the choice of an antidepressant. For potentially antagonists/agonists, the alpha2-adrenergic receptor antago- suicidal or highly impulsive patients, the SSRIs and the other nists, TCAs and related compounds, and MAOIs. The available newer agents would be a better initial choice than a cyclic com- formulations and their typical dosages are listed in Table 43-1, pound or an MAOI. Patients at elevated suicide risk who and aspects of their successful use are listed in Box 43-2. cannot tolerate these safer compounds or who do not respond to them should not receive large quantities or refillable pre- Selective Serotonin Re-uptake Inhibitors scriptions for TCAs or MAOIs. Generally, patients who are new to treatment or those at more than minimal risk for suicide or The overall efficacy of the SSRIs in the treatment of depression whose therapeutic relationship is unstable should receive a is equivalent to that of older agents, including the TCAs and limited supply of any . Evaluation for suicide risk the MAOIs moclobemide80 and phenelzine,81 while all SSRIs must continue even after the initiation of treatment. Although appear to be equally effective in the treatment of depression.82 suicidal thoughts are often among the first symptoms to However, there is some evidence suggesting that that the SSRI improve with antidepressant treatment, they may also be slow may be more effective than the remaining five to respond to treatment, and patients may become demoral- SSRIs in the treatment of MDD,83 although this evidence ized before therapeutic efficacy is evident. Side effects (such as remains controversial. agitation and restlessness) and, most important, inter-current Because of their favorable side-effect profile, the SSRIs are life events may exacerbate suicidal thoughts before a full thera- used as first-line treatment in the overwhelming majority of peutic response. Thus, rarely, for a variety of reasons, patients cases, with more than 90% of clinicians in one survey indicat- may temporarily become more suicidal following the initia- ing that SSRIs were their first-line treatment.84 Despite the tion of treatment. Should such worsening occur, appropriate tolerability and the widespread efficacy of the SSRIs, there is interventions may include management of side effects, more mounting evidence to suggest that depressed patients with frequent monitoring, discontinuation of the initial treatment, certain characteristics (including co-morbid anxiety disor- or hospitalization. In 2004, the FDA asked manufacturers of ders85 and a greater number of somatic symptoms [such as almost all the newer antidepressant drugs to include in their pain, headaches, and fatigue]86) respond less well to SSRIs labeling a warning statement that recommends close observa- than those without such characteristics.

TABLE 43-1A Available Preparations of Antidepressants: Selective Serotonin Re-uptake Inhibitors (SSRIs) Usual Daily Dose Extreme Dosage Plasma Levels Drug Therapeutic Dosage Forms (mg/day) (mg/day) (ng/ml) Fluoxetine (Prozac and generics) C: 10, 20, 40 mg 20–40 5–80 LC: 20 mg/5 ml Weekly: 90 mg (Luvox and generics)* T: 50, 100 mg 50–150 50–300 Paroxetine (Paxil and generics) T: 10, 20, 30, 40 mg 20–40 10–50 LC: 10 mg/5 ml CR: 12.5, 25, 37.5 mg 25–50 12.5–50 Sertraline (Zoloft and generics) T: 25, 50, 100 mg 50–150 25–300 LC: 20 mg/ml Citalopram (Celexa and generics) T: 10, 20, 40 mg 20–40 10–60 LC: 10 mg/5 ml Escitalopram (Lexapro and generics) T: 10, 20 mg 10–20 10–30 *Not marketed for depression in the US. C, Capsules; CR, controlled release; LC, liquid concentrate or solution; T, tablets.

Downloaded for Rohul Amin ([email protected]) at Uniformed Services Univ of the Health Sciences from ClinicalKey.com by Elsevier on September 21, 2018. For personal use only. No other uses without permission. Copyright ©2018. Elsevier Inc. All rights reserved. 494 PART X Treatment Approaches

TABLE 43-1B Available Preparations of Antidepressants: Serotonin Norepinephrine Re-uptake Inhibitors (SNRIs) Usual Daily Dose Extreme Dosage Plasma Levels Drug Therapeutic Dosage Forms (mg/day) (mg/day) (ng/ml) (Effexor and generics) T: 25, 37.5, 50, 75, 100 mg 75–300 75–450 XR: 37.5, 75, 150 mg Duloxetine (Cymbalta and generics) C: 20, 30, 60 mg 60–120 30–180 (Pristiq) T: 50, 100 mg 50–100 25–200 ER (Fetzima) C: 20, 40, 80, 120 mg 40–120 20–240 C, Capsules; T, tablets; XR, extended-release.

TABLE 43-1C Available Preparations of Antidepressants: Norepinephrine Re-uptake Inhibitors (NRIs) Usual Daily Dose Extreme Dosage Plasma Levels Drug Therapeutic Dosage Forms (mg/day) (mg/day) (ng/ml) Reboxetine* T: 4 mg 4–10 4–12 Atomoxetine (Strattera) T: 10, 18, 25, 40, 60 mg 40–80 40–120 *Not marketed for depression in the US. T, tablets.

TABLE 43-1D Available Preparations of Antidepressants: Serotonin Receptor Antagonists / Agonists Usual Daily Dose Extreme Dosage Plasma Levels Drug Therapeutic Dosage Forms (mg/day) (mg/day) (ng/ml) Trazodone (Desyrel and generics) T: 50, 100, 150, 300 mg 200–400 100–600 Trazodone Extended Release (Oleptro) T: 150, 300 mg 150–300 75–600 Nefazodone (Serzone and generics) T: 50, 100, 150, 200, 250 mg 200–400 100–600 Vilazodone (Viibryd) T: 10, 20, 40 mg 40–80 20–160 Vortioxetine (Brintellix) T: 5, 10, 20 mg 10–20 5–40 T, tablets.

TABLE 43-1E Available Preparations of Antidepressants: Norepinephrine Dopamine Re-uptake Inhibitors (NDRIs) Usual Daily Dose Extreme Dosage Plasma Levels Drug Therapeutic Dosage Forms (mg/day) (mg/day) (ng/ml) Bupropion (Wellbutrin and generics) T: 75, 100 mg 200–300 100–450 XR: 100, 150, 200 mg XL: 150, 300 mg Soltabs: 15, 30, 45 mg Soltabs, orally disintegrating tablet; T, tablets; XL, extended-release; XR, extended-release.

TABLE 43-1F Available Preparations of Antidepressants: Alpha2-receptor Antagonists Usual Daily Dose Extreme Dosage Plasma Levels Drug Therapeutic Dosage Forms (mg/day) (mg/day) (ng/ml) Mirtazapine (Remeron and generics) T: 15, 30, 45 mg 15–45 7.5–90 Soltabs: 15, 30, 45 mg Soltabs, orally disintegrating tablet; T, tablets.

Dosage and 100 mg for fluvoxamine (Luvox). Starting at lower doses and increasing the dose shortly thereafter (i.e., after 1 to 2 Because of their relatively low side-effect burden, the starting weeks)y ma further improve tolerability. Maximum therapeu- dose of SSRIs is often the minimally effective daily dose: tic doses for SSRIs are typically one-fold to four-fold greater 10 mg for escitalopram (Lexapro); 20 mg for fluoxetine than the starting dose. The dosages and formulations of the (Prozac), paroxetine (Paxil), and citalopram (Celexa); 50 mg SSRIs marketed in the US are listed in Table 43-1. Only one for sertraline (Zoloft); 25 mg for paroxetine CR (Paxil CR); of the SSRIs, fluvoxamine, is not approved for the treatment

Downloaded for Rohul Amin ([email protected]) at Uniformed Services Univ of the Health Sciences from ClinicalKey.com by Elsevier on September 21, 2018. For personal use only. No other uses without permission. Copyright ©2018. Elsevier Inc. All rights reserved. TABLE 43-1G Available Preparations of Antidepressants:Tricyclic Antidepressants (TCAs) and Other Cyclic Compounds Usual Daily Dose Extreme Dosage Plasma Levels Drug Therapeutic Dosage Forms (mg/day) (mg/day) (ng/ml) Imipramine (Tofranil and generics) T: 10, 25, 50 mg 150–200 50–300 >225* C: 75, 100, 125, 150 mg 43 INJ: 25 mg/2 ml Desipramine (Norpramin and generics) T: 10, 25, 50, 75, 100, 150 mg 150–200 50–300 >125 C: 25, 50 mg Amitriptyline (Elavil and generics) T: 10, 25, 50, 75, 100, 150 mg 150–200 50–300 >120† INJ: 10 mg/ml Nortriptyline (Pamelor and generics) C: 10, 25, 50, 75 mg 75–100 25–150 50–150 LC: 10 mg/5 ml Doxepin (Adapin, Sinequan, and generics) C: 10, 25, 50, 75, 100, 150 mg 150–200 25–300 100–250 LC: 10 mg/ml Trimipramine (Surmontil and generics) C: 25, 50, 100 mg 150–200 50–300 (Vivactil and generics) T: 5, 10 mg 10–40 10–60 Maprotiline (Ludiomil and generics) T: 25, 50, 75 mg 100–150 50–200 Amoxapine (Asendin and generics) T: 25, 50, 100, 150 mg 150–200 50–300 Clomipramine (Anafranil and generics) C: 25, 50, 75 mg 150–200 50–250 *Sum of imipramine plus desipramine. †Sum of amitriptyline plus nortriptyline. C, Capsules; INJ, injectable form; LC, liquid concentrate or solution; T, tablets.

TABLE 43-1H Available Preparations of Antidepressants: Monoamine Oxidase Inhibitors (MAOIs) Usual Daily Dose Extreme Dosage Plasma Levels Drug Therapeutic Dosage Forms (mg/day) (mg/day) (ng/ml) Phenelzine (Nardil and generics) T: 15 mg 45–60 15–90 Tranylcypromine (Parnate and generics) T: 10 mg 30–50 10–90 Isocarboxazid (Marplan and generics) T: 10 mg 30–50 30–90 Selegiline Transdermal System (patch) (Emsam) P: 6, 9, 12/day 6–12 6–18 T, tablets.

BOX 43-2 Requirements for Successful Use of Antidepressants

1. Good patient selection as determined by a thorough and with the expectation of tapering and discontinuing the comprehensive diagnostic evaluation. In particular, attention hypnotic when the depression has improved. Trazodone at should be paid to co-morbid psychiatric and medical lower doses (50–300 mg at bedtime) has been used in place disorders. of or other hypnotics to treat insomnia, 2. Choice of a drug with an acceptable side-effect profile for the particularly middle to late insomnia, in patients treated for given patient. depression with an SSRI. 3. Adequate dosage. In the absence of side effects and 8. Consideration of effects on sexual function. In particular, response, dose escalations within the recommended range decreased libido, delayed orgasm or anorgasmia, arousal should be pursued aggressively. difficulties, and have been reported with 4. Use of the antidepressant for at least 6–12 weeks to almost all of the classes of antidepressants. determine whether it is helping or not. 9. Awareness of co-morbid conditions. The co-morbid disorder 5. Consideration of drug side effects. Although there are some should influence initial treatment selection in choosing an differences in efficacy across the class of antidepressants for agent thought to be efficacious for the co-morbid condition, sutypes of depression, the major clinically significant as well as the depression, as with SSRIs and OCD or the differences among the antidepressants are in their side NDRI bupropion and attention-deficit disorder. effects. 10. Consideration of metabolic effects on drug levels and 6. Use of a drug that was clearly effective in the past if it was elimination half-life. The elderly may have alterations in hepatic well tolerated by the patient. metabolic pathways, especially so-called phase I reactions, 7. Selection of an appropriate agent for patients with initial which include demethylation and hydroxylation, which are insomnia, for example, a sedating secondary amine TCA (e.g., involved in the of both SSRIs and cyclic nortriptyline) given at bedtime (a strategy used by some antidepressants. In addition, renal function may be decreased, clinicians), avoidance of agents with anticholinergic and and there may be increased end-organ sensitivity to the cardiovascular side effects (therefore, the sleep-enhancing, effects of antidepressant compounds. Because the elimination

alpha2-adrenergic receptor antagonist mirtazapine would be half-life of antidepressants can be expected to be significantly preferred, with the expectation that daytime sedation will greater than what it is in younger patients, accumulation of abate over time with these medications). An alternative to active drug will be greater and occur more slowly. Clinically prescribing a sedating antidepressant is the temporary use of this means that the elderly should be started on lower doses, a short-acting or other hypnotic combined that dosage increases should be slower, and that the ultimate with an SSRI or another non-sedating newer antidepressant, therapeutic dose may be lower than in younger patients.

NDRI, Norepinephrine/dopamine re-uptake inhibitor; OCD, obsessive-compulsive disorder; SSRI, selective serotonin re-uptake inhibitor.

Downloaded for Rohul Amin ([email protected]) at Uniformed Services Univ of the Health Sciences from ClinicalKey.com by Elsevier on September 21, 2018. For personal use only. No other uses without permission. Copyright ©2018. Elsevier Inc. All rights reserved. 496 PART X Treatment Approaches of depression in the US, as it is approved only for the treat- dizziness, insomnia, nervousness, irritability, , and ment of OCD, although several placebo-controlled trials have agitation.115,116 The risk of such adverse events occurring seems demonstrated the efficacy of fluvoxamine in MDD.87,88 to be inversely related to the half-life of the SSRI, with fluoxe­ tine reported as having a significantly lower risk than paroxe­ Side-effect Profile tineo in tw studies.115,117 For more severe discontinuation-related adverse events, re-institution of the SSRI and slow taper may The most common side effects of the SSRIs are nausea, tremor, 118 excessive sweating, flushing, headache, insomnia, activation or be necessary to alleviate these symptoms. sedation, jitteriness, dizziness, rash, and dry mouth.89 Sedation Drug Interactions does not appear to occur more often with any particular SSRI, while the SSRIs appear to be equally well tolerated and effective With the exception perhaps of citalopram, and its stereo­isomer in the treatment of depressed patients, regardless of whether escitalopram,119 ySSRIs ma inhibit cytochrome P (CYP) 450 they cause insomnia, activation, or sedation.82 The use of SSRIs isoenzymes to varying degrees, potentially causing substrate is also associated with the emergence of levels to rise, or reducing conversion of a substrate into its (including decreased libido, , impotence, active form. Concern about drug interactions, however, is per- and anorgasmia), or the worsening of pre-existing sexual tinent to patients who take medications with narrow thera- dysfunction in depression.90 These side effects tend peutic margins that are metabolized by isoenzymes inhibited to improve rapidly after temporary (“drug holiday”) discon- by an SSRI and if the prescriber is unfamiliar with or unable tinuation of the SSRIs, particularly those SSRIs with a shorter to determine the appropriate dose adjustment. Given their vast half-life,91 although prolonging such drug holidays carries a risk availability, reports of clinically significant interactions with of withdrawal effects and of depressive relapse. Some patients the SSRIs are remarkably rare. Among the SSRIs, fluvoxamine treated with SSRIs may also experience cognitive symptoms is a potent CYP 1A2 and CYP 2C19 inhibitor, and a moderate (such as mental slowing and worsened attention), psychologi- CYP 2C9, CYP 2D6, and CYP 3A4 inhibitor, while fluoxetine cal symptoms (such as apathy and emotional blunting),92 and and paroxetine are potent CYP 2D6 inhibitors, and fluoxetine’s motor symptoms (such as bruxism and akathisia).93,94 ,Other main metabolite, norfluoxetine, has a moderate inhibitory less common, adverse events associated with SSRI treatment effectYP on C 3A4.119 Sertraline is a moderate CYP 2D6 inhibi- include diarrhea, tremor, bruxism, rash, hyponatremia, hair tor, while citalopram and escitalopram appear to have little loss, and the syndrome of inappropriate antidiuretic hormone effect on the major CYP isoforms.119 However, for all of the (SIADH) secretion. There are also case reports of the SSRIs SSRIs, some vigilance is reasonable concerning the possibility worsening motor symptoms in patients with Parkinson’s of increased therapeutic or toxic effects of other co-prescribed disease,95–97 as well as creating increased requirements for levo- drugs metabolized by P450 2D6. In particular, if combining dopa in Parkinson’s patients following initiation of an SSRI for aCA T with an SSRI, the TCA should be initiated with low depression.98 SSRIs have been associated with abnormal bleed- doses, and plasma levels should be monitored. Given the high ing (e.g., bruising and epistaxis) in children and adults who capacity of the CYP 450 3A3/3A4 system, inhibition of this have unremarkable routine hematological laboratory results, isoenzyme is not a major concern for the SSRIs, although except for abnormal bleeding time or platelet counts.99 A sys- fluvoxamine, and less so fluoxetine, can inhibit it to some tematic study of this issue has failed to reveal abnormalities in extent. Of little importance to drug interactions is the high platelet aggregation, hematopoiesis, or coagulation profile in rate of protein-binding of the SSRIs because, if other drugs are SSRI-treated patients.100 Although many patients may also displaced from carrier proteins, the result is simply an increase experience reduced appetite and weight loss during the acute in the rate and amount of free drug being metabolized. The phase of treatment with SSRIs,101,102 yan beneficial effects of the augmentation and combination of SSRIs with other seroton- SSRIs with respect to weight loss do not seem to be sustained ergic agents, , 5-HT, or MAOIs may also result in during the continuation and maintenance phases of treat- the serotonin syndrome; SSRIs should never be used concomi- ment,101,102 while one study reveals a greater risk for significant tantly with MAOIs, because there have been a number of weight gain during long-term treatment with paroxetine, but reports of fatal cases of serotonin syndrome due to the simul- not fluoxetine or sertraline.103 Although SSRI-induced side taneous use of these classes of drugs or to the inadequate effects appear to be well tolerated by most patients,104 for washout period between the two.54–56 depressed patients who are unable to tolerate one SSRI, switch- ing to another SSRI has been effective and well tolerated in Use of Selective Serotonin Re-uptake Inhibitors in most. cases 105–108 For patients complaining of GI side effects Pregnancy and the Post-partum Period with paroxetine, the continued-release formulation (Paxil CR), reported to have a lower incidence of nausea, may be used in There is accumulating information about the use of SSRIs in place of the standard formulation.109 As with other antidepres- pregnancy, although the bulk of the available data are on fluoxetine. One prospective study of 128 pregnant women sants, the potential adverse neuroendocrine and skeletal effects 120 of the SSRIs have yet to be systematically explored.110 However, who took fluoxetine, 01 to 80 mg/day (mean 25.8 mg), a study from our group has shown that 4.5% of men and 22.2% during their first trimester did not find elevated rates of major of women with MDD developed new-onset hyperprolactine- malformations compared with matched groups of women mia following SSRI treatment,111 and daily SSRI use in adults taking TCAs or drugs thought not to be teratogenic. There was 50 years and older was associated with a two-fold increased risk a, higher albeit not statistically significant, rate of miscarriages of fractures after adjustment for potential co-variates.112 The in the fluoxetine (13.5%) and TCA (12.2%) groups compared SSRIs also appear to possess extremely low toxicity in over- with the women exposed to known non-teratogenic drugs dose.113 ,Finally of all antidepressants, fluoxetine has the most (6.8%). Whether this increased rate of miscarriages is biologi- extensive literature supporting its reproductive safety.114 cally significant and, if so, whether it relates to the drugs or to the depressive disorder could not be determined from this Selective Serotonin Re-uptake Inhibitors study. Decisions on continuing antidepressant drugs during Discontinuation Syndrome pregnancy must be individualized, but it must be recalled that the effects of severe untreated depression on maternal and A number of reports also describe discontinuation-emergent fetal health may be far worse than the unknown risks of fluox- adverse events after abrupt cessation of SSRIs, including etine or tricyclic drugs. A large registry of fluoxetine exposure

Downloaded for Rohul Amin ([email protected]) at Uniformed Services Univ of the Health Sciences from ClinicalKey.com by Elsevier on September 21, 2018. For personal use only. No other uses without permission. Copyright ©2018. Elsevier Inc. All rights reserved. Antidepressants 497 during pregnancy is consistent with generally reassuring data discontinuation-related adverse events similar to those fromCA the T era that antidepressant agents are not evidently described for the SSRIs.145 ,Finally in one uncontrolled study, 43 teratogens. 4 of 13 patients treated with venlafaxine during electroconvul- On the other hand, infants exposed to SSRIs during late sive therapy (ECT) experienced asystole.146 Although the pregnancy may be at increased risk for serotonergic CNS authors noted that this serious adverse event only occurred in adverse effects, although the incidence of these events has not patients on daily doses of venlafaxine greater than 300 mg and been well established. Recently, the FDA has issued a warning in patients anesthetized with propofol, in the absence of for all SSRIs, reporting an increased risk for neonatal toxicity further data the use of venlafaxine in patients requiring ECT and recommending cessation of treatment before delivery. and perhaps even general anesthesia should be avoided. However, in clinical practice, the risk of post-partum depres- Desvenlafaxine (Pristiq) is a major of sion often warrants continued treatment and close monitoring venlafaxine and an SNRI with little affinity for muscarinic, of the newborn. histaminic, or adrenergic receptors. Desvenlafaxine is metabo- Whenever possible, unnecessary exposure to any drug lizedyYP b C 450 3A4 and it is a very weak inhibitor of 2D6. should be minimized, and thoughtful pre-pregnancy treat- The half-life of desvenlafaxine is about 10 hours. The augmen- ment planning and consideration of alternative interventions, tation and combination of desvenlafaxine with other seroton- such as psychotherapies (e.g., cognitive-behavioral therapy ergic, agents tryptophan, 5-HT, or MAOIs may also result in the [CBT]), are to be recommended. serotonin syndrome. Desvenlafaxine was found to be more SSRIs are secreted in breast milk. Because their effects on effective than placebo in the treatment of MDD in two separate normal growth and development are unknown, breast-feeding randomized, double-blind clinical trials. Side effects reported should be discouraged for mothers who are on SSRIs. in those studies included nausea, dry mouth, sweating, som- nolence, anorexia, constipation, asthenia, , tremor, Serotonin Norepinephrine Re-uptake Inhibitors nervousness, abnormal vision, and sexual dysfunction. There have been reports of increases in diastolic , and Venlafaxine, duloxetine, desvenlafaxine, and levomilnacipran the abrupt discontinuation of desvenlafaxine also carries a risk share the property of being relatively potent re-uptake inhibi- of discontinuation-related adverse events similar to those tors of serotonin and norepinephrine and are therefore con- described for the SSRIs. There are also reports of an increase in sidered SNRIs. They are all approved for the treatment of liver enzymes among patients treated with desvenlafaxine. depression in the US. Desvenlafaxine is commonly used at doses between 50 and Venlafaxine and Desvenlafaxine 100 mg daily in a single administration. Venlafaxine (Effexor) was the first SNRI to gain FDA approval Duloxetine for the treatment of depression. At daily doses greater than 121 Duloxetine (Cymbalta) also inhibits the re-uptake of both 150 mg, venlafaxine inhibits the re-uptake of both sero­ 147 tonin and norepinephrine, while mostly inhibiting the serotonin and norepinephrine. Duloxetine appears to be as 122,123 effective as the SSRIs in the treatment of MDD, although in re-uptake of serotonin at lower doses. The augmentation 148 and combination of venlafaxine with other serotonergic more severe depression there may be some advantages. agents, tryptophan, 5-HT, or MAOIs may also result in the Duloxetine lacks significant cholinergic, antihistaminergic, serotonin syndrome. Venlafaxine lacks significant cholinergic, and alpha1-adrenergic blocking effects. Duloxetine is exten- sively metabolized to numerous metabolites that are primarily antihistaminergic, and alpha1-adrenergic–blocking effects. Venlafaxine is metabolized by CYP 450 2D6, of which it is excreted into the in the conjugated form. The major also a very weak inhibitor. The half-lives of venlafaxine and its metabolites in plasma are glucuronide conjugates of 4-hydroxy active metabolite O-desmethylvenlafaxine are about 5 and 11 duloxetine (M6), 6-hydroxy-5-methoxy duloxetine (M10), hours, respectively. The drug and this metabolite reach steady 4,6-dihydroxy duloxetine (M9), and a sulfate conjugate of state in plasma within 3 days in healthy adults. Venlafaxine is 5-hydroxy-6-methoxy duloxetine (M7). Duloxetine is metabo- generally effective at daily doses at or above 150 mg, and is lizedyYP b C 450 2D6, of which it is also a moderate inhibitor, often started at 75 mg or even 37.5 mg, typically in its intermediate between paroxetine and sertraline. Drugs that extended-release (XR) formulation.124 Several meta-analyses inhibit this enzyme may increase duloxetine concentrations. suggest venlafaxine to be more effective than the SSRIs in the The half-life of duloxetine is about 12.5 hours. Abrupt discon- treatment of MDD,125–127 with the exception of escitalopram, tinuation of duloxetine is associated with a discontinuation- 83 emergent adverse event profile similar to that seen with SSRIs whose relative efficacy appears to be comparable. 149 Venlafaxine, along with the SSRIs and bupropion, is also and SNRI antidepressants. , Therefore duloxetine’s discon- commonly chosen as a first-line treatment for depression.84 tinuation should be accomplished with a gradual taper (over Venlafaxine is also used in treatment-refractory depression as at least 2 weeks) to reduce the risk of discontinuation-emergent a “next-step” strategy,128–135 reported as the most popular adverse events. Duloxetine is commonly used at daily doses switch strategy for refractory depression in one large survey of of 60 to 120 mg, often started at 30 mg. Duloxetine also 136 appears to be effective in the treatment of somatic symptoms clinicians. 150,151 Common side effects of venlafaxine include nausea, insom- of depression, such as pain. Common side effects associ- ated with duloxetine include dry mouth, headache, nausea, nia, sedation, sexual dysfunction, headache, tremor, palpi­ 152 137 somnolence, sweating, insomnia, and fatigue. Duloxetine tations, and dizziness, as well as excessive sweating, 152 tachycardia, and . There are also reports of does not appear to cause hypertension. bruxism.138,139 Venlafaxine’s potential for sexual dysfunction appears to be comparable to that of the SSRIs.140,141 The inci- Levomilnacipran and dence of GI side effects and dizziness appears to be lower with Levomilnacipran (Fetzima) is an SNRI and is the 1S-2R enan- the use of the XR formulation than the immediate-release tiomer of milnacipram, an SRNI approved for depression in formulation.142 Between 2% and 6% of patients also experi- Europe (brand names: Dalcipran, Ixel). Levomilnacipran is ence an increase in diastolic blood pressure,143 which appears metabolized by CYP 450 3A4 and it is not an inhibitor of CYP to be dose-related.144 The abrupt discontinuation of ven­ 450 systems. Its half-life is about 12 hours. The augmentation lafaxine, given its short half-life, also carries a risk of and combination of milnacipran or levomilnacipran with

Downloaded for Rohul Amin ([email protected]) at Uniformed Services Univ of the Health Sciences from ClinicalKey.com by Elsevier on September 21, 2018. For personal use only. No other uses without permission. Copyright ©2018. Elsevier Inc. All rights reserved. 498 PART X Treatment Approaches other serotonergic agents, tryptophan, 5-HT, or MAOIs may Atomoxetine has also been associated with mild increases in also result in the serotonin syndrome. A number of studies blood pressure and pulse that plateau during treatment and had demonstrated that the SNRI milnacipran153 is equivalent resolve after discontinuation. There have been no effects seen to the SSRIs154–157 and the TCAs158–162 and superior to placebo on the QT interval. It is a substrate of CYP 2D6 and its in the treatment of depression.163,164 ,Similarly several studies biotransformation involves aromatic ring hydroxylation, ben- have shown that the norepinephrine and serotonin re-uptake zylic oxidation, and N-demethylation. At high therapeutic inhibitor levomilnacipran is superior to placebo in the treat- doses, atomoxetine inhibits CYP 2D6 and CYP 3A activity, ment of depression.165–167 Because of the potent norepine- although in vivo studies clearly indicate that atomoxetine phrine re-uptake inhibition, levomilnacipran appears to be administration with substrates of CYP 2D6 and CYP 3A does particularly effective in treating fatigue in depression. Common not result in clinically significant drug interactions. side effects reported during treatment with milnacipran include headaches, dry mouth, dysuria, tremor, tachycardia, weight gain, and sedation, although the incidence of weight Norepinephrine/Dopamine Re-uptake Inhibitors gain and sedation with milnacipran is lower than with the TCAs.162 Levomilnacipran’s frequently reported adverse events Bupropion (≥ 5% in levomilnacipran ER and twice the rate of placebo) The NDRI bupropion appears to be as effective as the SSRIs are nausea, dizziness, constipation, tachycardia, urinary hesi- in the treatment of depressive177,178 and anxiety symptoms in tation, , insomnia, vomiting, and elevated blood depression,179 and more effective than the SSRIs in the pressure. Daily doses of milnacipran range from 50 to 200 mg, treatment of sleepiness and fatigue in depression.180 Interest- often divided in twice-daily dosing, whereas the usual dose of ingly, bupropion has been shown to be as effective as the SNRI levomilnacipran ER is 40–120 mg once/daily venlafaxine in the treatment of MDD.181 Bupropion is a compound that is effective Norepinephrine Re-uptake Inhibitors for the treatment of MDD. Bupropion is structurally related to amphetamine and the sympathomimetic diethylpropion, and Reboxetine it primarily blocks the re-uptake of dopamine and norepine- Reboxetine168 yacts b selectively inhibiting the norepinephrine phrine and has minimal or no affinity for post-synaptic recep- transporter, thereby increasing synaptic norepinephrine levels. tors. Although some researchers have argued that the NDRI Reboxetine, a morpholine compound, is chemically unrelated bupropion’s effect on norepinephrine is primarily through an to the other antidepressants. It is highly protein-bound and increase in pre-synaptic release, there is still convincing evi- has a plasma half-life of about 13 hours. The drug does not dence for binding of both norepinephrine and dopamine appear to be a meaningful inhibitor of the CYP 450 system transporters. Bupropion is rapidly absorbed after oral admin- and is metabolized itself by CYP 450 3A4 isozyme with two istration and demonstrates biphasic elimination, with an inactive metabolites. Increased blood pressure has been elimination half-life of 11 to 14 hours. It is converted to three reported to be a problem for some patients, particularly those active metabolites, , threohydrobupropion, with a genetic variant of the norepinephrine transporter and erythrohydrobupropion, all of which have been demon- (SCL6A2). Reboxetine has not received FDA approval for the strated to have antidepressant activity in animal models. treatment of depression, but is available in Europe for the Bupropion lacks anticholinergic properties and does not cause treatment of depression (brand name: Edronax). Double- postural hypotension or alter cardiac conduction in a clini- blind, placebo-controlled trials suggest reboxetine to be more cally significant manner. It is a substrate of CYP 450 2B6 and effective than placebo169–172 and as effective as fluoxetine170 in appears to have CYP 450 2D6 inhibition potential, which the treatment of MDD. The starting daily dose is usually 8 mg suggests that, when it is combined with fluoxetine or paroxet- but can be as low as 4 mg, with effective daily doses ranging ine, both 2D6 substrates, levels of the SSRI may increase. One advantage of treatment with bupropion compared to the SSRIs from0 8 to 1 mg given in divided doses (twice a day). Common 140,141,182–187 side effects include insomnia, headache, dry mouth, diaphore- is the lower risk of sexual dysfunction. Treatment 170 with bupropion is also associated with a lower incidence of sis, and constipation, as well as urinary hesitancy. The inci- 185,186 170 173 GI side effects (e.g., nausea and diarrhea) and seda- dence of nausea, headache, fatigue, and sexual dysfunction 177,179,185 appears to be more common during treatment with the SSRIs tion than the SSRIs. Although a difference in terms of than with reboxetine. The urinary hesitancy and constipation weight changes in bupropion and SSRI-treated depressed patients is not immediately apparent during the acute phase do not appear to reflect anticholinergic, but rather noradren- 183,185,186,188–190 ergic, effects. of treatment in randomized trials, there is evi- dence to suggest that any beneficial effects of SSRIs in terms Atomoxetine of weight reduction during the acute phase are not sustained during the continuation and maintenance phases.101,102 In fact, Atomoxetine also selectively inhibits the re-uptake of nor­ long-term treatment with some SSRIs may also result in long- 174 epinephrine. In vitro, ex vivo, and in vivo studies have shown term weight gain.103 In contrast, long-term (44 weeks) bupro- that atomoxetine is a highly selective antagonist of the pre- pion treatment appears to result in weight changes no different synaptic norepinephrine transporter, with little or no affinity than those of placebo in MDD.191 ,Thus long-term treatment for other noradrenergic receptors or other neurotransmitter with bupropion may carry a lower risk of weight gain than transporters or receptors, with the exception of a weak affinity long-term treatment with the SSRIs. for the serotonin transporter. Atomoxetine is rapidly absorbed, The dose range for the sustained-release (SR) formulation with peak plasma concentrations occurring 1 to 2 hours after of bupropion (Wellbutrin SR) is 150 to 450 mg in twice a day dosing, and its half-life hovers around 3 to 4 hours. While or three times a day dosing, with 100 or 150 mg being a atomoxetine is an FDA-approved treatment for attention- common starting dose. A once-daily dose formulation (Well- deficit/hyperactivity disorder (ADHD) (brand name: Strat- butrin XL), available in 150 and 300 mg doses, was subse- tera), there is a single open trial of atomoxetine in depression quently introduced. Common side effects include agitation, involving0 1 patients, with daily doses ranging from 40 to insomnia, weight loss, dry mouth, headache, constipation, and 175 70 mg. Common side effects reported so far include tremor.189 The major medically important adverse event associ- 176 decreased appetite, insomnia, and increased blood pressure. ated with bupropion is seizure. With the immediate-release

Downloaded for Rohul Amin ([email protected]) at Uniformed Services Univ of the Health Sciences from ClinicalKey.com by Elsevier on September 21, 2018. For personal use only. No other uses without permission. Copyright ©2018. Elsevier Inc. All rights reserved. Antidepressants 499 formulation the rate is 0.4% (4 per 1,000) at doses up to effective antidepressant dosage in the 450 to 600 mg/day 450 mg/day, whereas with bupropion SR the rate is of 0.1% range in divided doses. Slower dose titration is recommended 43 (1 per 1,000) at doses up to the target antidepressant dose of in the elderly. Nefazodone has been found to inhibit CYP 3A4 300y mg/da (Wellbutrin SR Prescribing Information).192 SSRI and to result in serotonin syndrome when combined with antidepressants are also associated with seizures at a similar SSRIs. Common side effects include somnolence, dizziness, rate of approximately 0.1% (Wellbutrin SR Prescribing Infor- dry mouth, nausea, constipation, headache, amblyopia, and mation). Patients should only be administered bupropion blurred vision.202 An unusual but occasional adverse effect is with extreme caution if a predisposition to seizure is present. irritability (possibly related to its mCPP metabolite, which For this reason, the maximum daily dose for bupropion SR may occur in higher levels in the presence of a CYP 450 2D6 and bupropion XL is 450 mg, with no single dose above inhibitor). Treatment with nefazodone has the advantage of a 200 mg for the SR formulation. In addition, bupropion may lower risk of long-term weight gain than the SSRIs or TCAs,97 be more likely to induce seizures in patients with bulimia perhaps because of the appetite-reducing effects of mCPP.203 nervosa and histories of head trauma and it should not be Nefazodone also has the advantage of a lower risk of sexual used in these patients. Since the risk of seizure appears to be side effects than the SSRIs.140,141,204 A rare but serious dose-related and related to the peak plasma concentrations, of nefazodone is priapism of both the penis and clitoris,205,206 the SR and XL formulations are thought to be associated with which requires immediate medical attention. In addition, an a somewhat lower seizure risk, estimated at 0.1% for daily increasing number of reports suggest that treatment with doses lower than 450 mg.192 nefazodone is associated with an increase risk of hepatotoxic- ity (approximately 29 cases per 100,000 patient years),207 often Serotonin Receptor Antagonists/Agonists severe (more than 80% of cases), and often appearing during the first 6 months of treatment.208 o,T date there has even been Trazodone one reported death due to such hepatotoxicity.209 , Therefore this agent should be avoided in patients with current or a Although the serotonin receptor antagonists trazodone history of liver abnormalities; liver enzymes should be checked (Desyrel) and nefazodone (Serzone) have been shown to be as periodically in patients on nefazodone. The minimal effective effective as the SSRIs in the treatment of depression,193 they are doses for nefazodone are usually 300 mg daily, with 600 mg used extremely infrequently as monotherapy for depression.84 daily being the optimal dose. Trazodone is rapidly absorbed following oral administration, achieving peak levels in 1 to 2 hours. It has a relatively short elimination half-life of 3 to 9 hours and is excreted mainly in Vilazodone urine (75%); its metabolite mCPP has a similar pharmacoki- Vilazodone (Vibryd) is a serotonin 5-HT receptor partial netic profile. Despite the short half-life, once-daily dosing at 1A agonist and a selective serotonin re-uptake inhibitor. Studies bedtime is the usual route of administration because of its have shown its superiority over placebo in the treatment of sedating properties. When used as monotherapy for the treat- depression.210 The augmentation and combination of vilazo- ment of depression, a patient is typically started on 100 to done with other serotonergic agents, tryptophan, 5-HT, or 150 mg daily either in divided doses or in a single bedtime dose MAOIsy ma also result in the serotonin syndrome. Vilazodone and gradually increased to 200 to 300 mg/day. An extended- lacks significant cholinergic, antihistaminergic, and alpha - release formulation of trazodone (Oleptro) is available. The 1 adrenergic–blocking effects. Vilazodone is metabolized prim­ dose range for the extended-release is between 150 and 300 mg arilyyYP b C 450 3A4 and is a moderate inhibitor of 2C19 and qHS. For optimal benefit, doses in the range of 400 to 600 mg 2D6. The terminal half-life of vilazodone is about 25 hours. may be needed for either formulation. Low-dose trazodone (25 Vilazodone therapy is typically initiated at a dosage of 10 mg to0 15 mg at bedtime) is commonly used in the treatment of once daily and incrementally adjusted over 14 days to the insomnia secondary to antidepressant use,194 a strategy that recommended target daily dose of 40 mg; for optimal bio­ may also result in an improvement in depressive symptoms.195 availability and effectiveness, it should be taken after a light The most common side effects of trazodone are sedation, or high-fat meal. The adverse effects most commonly reported orthostatic hypotension, and headaches. Trazodone lacks the in clinical trials of vilazodone were diarrhea, nausea, vomit- -like properties of the cyclic antidepressants but has ing, dizziness, insomnia and dry mouth. Treatment-related been associated in rare cases with cardiac arrhythmias, which sexual side effects may be less likely than with SSRIs: in three may be related to trazodone’s ability to inhibit potassium chan- placebo-controlled studies, 8.0% of vilazodone-treated nels., Thus trazodone should be used with caution in patients patients and 0.9% of placebo-treated patients reported ≥ 1 with known cardiac disease. A rare but serious side effect of sexual-function-related treatment-emergent adverse event (P< trazodone is priapism of both the penis and clitoris,196,197 which 0.001)).211 Vilazodone was not associated with clinically- requires immediate medical attention. Priapism has been relevant weight change in the short-term trials. In an open- attributed to the alpha-adrenoceptor blocking properties of label 1-year study of vilazodone, mean weight increased by trazodone by interference with the sympathetic control of 1.7 kg among the observed cases.212 penile detumescence.198 Rare cases of hepatotoxicity have been associated with the use of trazodone,199 and fatal cases of tra- zodone overdose have also been reported.200 In one review, Vortioxetine trazodone was reported to carry one of the lowest risks for Vortioxetine (Brintellix) is a recently-approved antidepressant seizure of all antidepressants examined.201 The minimal effec- with multi-modal activity that functions as a serotonin 5-HT , tive dose for trazodone is usually 300 mg daily. 3 5-HT7 and 5-HT1D receptor antagonist, serotonin 5-HT1B recep- Nefazodone tor partial agonist, serotonin 5-HT1A receptor agonist and inhibitor of the serotonin transporter in vitro. Vortioxetine has Nefazodone has less affinity for the alpha1-adrenergic receptor shown to be able to increase extracellular serotonin, dopamine, and is therefore less sedating. The half-life of nefazodone is and norepinephrine levels in the medial prefrontal cortex and approximately 5 hours. The usual starting dosage is 50 mg/ ventral hippocampus. The augmentation and combination of day given at bedtime or twice a day, titrated up in the absence vortioxetine with other serotonergic agents, tryptophan, 5-HT, of daytime sedation as rapidly as tolerated to achieve a usually or MAOIs may result in the serotonin syndrome. Vortioxetine

Downloaded for Rohul Amin ([email protected]) at Uniformed Services Univ of the Health Sciences from ClinicalKey.com by Elsevier on September 21, 2018. For personal use only. No other uses without permission. Copyright ©2018. Elsevier Inc. All rights reserved. 500 PART X Treatment Approaches

lacks significant cholinergic, anti-histaminergic, and alpha1- administered drugs are hardly affected by mirtazapine. Mirta- adrenergic–blocking effects. Vortioxetine is metabolized prim­ zapine is associated with more sedation and weight gain than arilyyYP b C 450 2D6. The terminal half-life of vortioxetine is the. SSRIs 233–236 The widespread use of mirtazapine as a first- about 66 hours. Vortioxetine therapy is typically initiated at a line agent in depression has been primarily limited by its dosage0 of 1 mg once daily and incrementally adjusted 20 mg sedative effects and weight gain.237 In addition to sedation and once, daily if necessary; the dose can be lowered to 5 mg daily, weight gain, common side effects associated with mirtazapine if necessary based on tolerability issues. Nausea, dry mouth, include dizziness, dry mouth, constipation, and orthostatic diarrhea, nasopharyngitis, headache, dizziness, somnolence, hypotension. Due to blockade of 5-HT2 and 5-HT3 ,receptors vomiting, dyspepsia, constipation and fatigue were reported mirtazapine is associated with a lower risk of headaches233 in ≥ 5% of patients receiving vortioxetine.213 Rates of treatment- and nausea233–235,238,239 than the SSRIs. Treatment with mirta- emergent sexual dysfunction (TESD) in the vortioxetine dosing zapine is also associated with a lower incidence of sexual groups were similar to placebo.213 In a 52-week open-label dysfunction than the SSRIs.141,234 In addition, switching to extension study, vortioxetine was associated with minimal mirtazapine may alleviate SSRI induced sexual dysfunction in weight gain.214 Recently presented data suggest that vortioxet- SSRI-remitters.240 Severe neutropenia has been rarely reported iney ma have the additional advantage of distinctive pro- (1 in 1,000) with an uncertain relationship to the drug, but cognitive effects. as with other psychotropics, the onset of infection and fever should prompt the patient to contact his or her physician. The drug is most efficacious at doses of 30 to 45 mg (although 60y mg/da has been used in refractory cases) usually given in Ritanserin, a serotonin 5-HT and 5-HT antagonist, is not 2A 2C a single bedtime dose. Available in 15, 30, and 45 mg scored FDA-approved, but is available in Europe. One placebo- tablets and in an orally soluble tablet formulation (Soltab) at controlled study revealed ritanserin to be effective in the treat- 15, 30, and 45 mg, the lower dose may be sub-optimal, and ment of dysthymic disorder,215 while a separate study found compared with 15 mg, the 30 mg dose also may be less or at ritanserin to be as effective as amitriptyline in patients with least not more sedating, possibly as a consequence of the depression and chronic headaches.216 Ritanserin appears to be noradrenergic effects being recruited at that dose. The starting effective for depression at doses above 5 mg. daily dose can be as low as 7.5 mg in the elderly. Agomelatine Mianserin Agomelatine, a newer agent, is a selective 5-HT antagonist 2C Mianserin is approved for depression in Europe (brand name: and acts as a melatonin MT and MT receptor agonist. To date, 1 2 Lantanon) and is not FDA-approved. Double-blind studies ato least tw placebo-controlled trials have found agomelatine report the efficacy of mianserin in the treatment of MDD to (25 mg) to be more effective than placebo and as effective as be equivalent to the TCAs.241–243 The most common side effects the SSRIs in the treatment of MDD,217–219 and as effective as include somnolence, weight gain, dry mouth, sleep problems, the SSRI paroxetine in MDD.217 Agomelatine is approved for tremor, and headaches.244 eEffectiv daily doses for mianserin the treatment of depression in Europe, but not in the US. range from 30 to 60 mg, usually given at bedtime. Buspirone and Gepirone Buspirone (BuSpar) and gepirone (Ariza) act as full agonists Tricyclic and Related Cyclic Antidepressants at serotonin 5-HT autoreceptors and are generally, but not 1A Oral preparations of TCAs and related drugs are rapidly and exclusively, partial agonists at post-synaptic serotonin 5-HT 1A completely absorbed from the GI tract; a high percentage of receptors.29 Buspirone is FDA-approved as a treatment for an oral dose is metabolized by the liver as it passes through anxiety and not for depression, while gepirone has not been the portal circulation (first-pass effect). The TCAs are metabo- FDA-approved. Nevertheless, a number of double-blind trials lizedy b the microsomal enzymes of the liver; the tertiary report buspirone220–223 and gepirone224–228 to be more effective amines are first monodemethylated to yield compounds that than placebo in the treatment of MDD. One advantage of are still active. Indeed, the desmethyl metabolites of amitriptyl- gepirone and perhaps buspirone is that their use does not ine and imipramine are nortriptyline and desipramine, respec- appear to be related to a greater incidence of weight gain or tively, and are marketed as antidepressants. Other major sexual side effects than placebo, at least during the acute phase metabolic pathways include hydroxylation (which may yield of treatment of depression.228 eEffectiv doses for buspirone partially active compounds) and conjugation with glucuronic and gepirone for depression range between 30 and 90 mg and acid to produce inactive compounds. TCAs are highly 20 and 80 mg, respectively. Side effects are similar for these lipophilic, meaning the free fraction passes easily into the two agents and include headache, dizziness, light-headedness, brain and other tissues. They are also largely bound to plasma nausea, and insomnia.222,229 proteins. Given their lipophilicity and protein-binding, they are not removed effectively by hemodialysis in cases of over- Alpha2-Adrenergic Receptor Antagonists dose. The time course of metabolism and elimination is Mirtazapine biphasic, with approximately half of a dose removed over 48 to 72 hours and the remainder, strongly bound to tissues and Mirtazapine is as effective as the SSRIs230 and venlafaxine231,232 plasma proteins, slowly excreted over several weeks. There is in the treatment of MDD. Mirtazapine shows linear pharma- considerable variation among individuals in their metabolic cokinetics over a dose range of 15 to 80 mg and its elimina- rate for cyclic antidepressants based on genetic factors, age, tion half-life ranges from 20 to 40 hours, consistent with its and concomitantly taken drugs. In fact, when metabolic dif- time to reach steady state (4 to 6 days). Biotransformation is ferences are combined with variation in the degree of protein- mainly mediated by the CYP 2D6 and CYP 3A4 isoenzymes. binding, as much as a 300-fold difference in effective drug Inhibitors of these isoenzymes, such as paroxetine and fluox- levelsy ma be found among individuals. etine, cause modestly increased mirtazapine plasma concen- Although TCAs’ overall efficacy in treating depression is trations, while mirtazapine has little inhibitory effects on equivalent to that of the SSRIs,245 they tend to have considerably CYP isoenzymes; therefore, the of co- more side effects and, due to their ability to block the

Downloaded for Rohul Amin ([email protected]) at Uniformed Services Univ of the Health Sciences from ClinicalKey.com by Elsevier on September 21, 2018. For personal use only. No other uses without permission. Copyright ©2018. Elsevier Inc. All rights reserved. Antidepressants 501 aforementioned receptors, as well as the sodium channel,246 Amoxapine is the only TCA with documented, significant 247 248 49 TCAs are often arrythmogenic and epileptogenic when dopamine D2 receptor antagonism. , Therefore there have 43 taken in very large (supra-therapeutic) quantities. As a result, been case reports of tardive dystonia and dyskinesia associated they are rarely chosen as first-line agents in the treatment of with amoxapine treatment,257,258 and amoxapine should be depression.84 Furthermore, several studies also suggest that avoided in patients with co-morbid depression and Parkin- TCAsy ma be more effective than the SSRIs in the treatment of son’s disease.259 melancholic depression, or in the treatment of depressed Anticholinergic-related side effects result from the affinity patients with certain co-morbid medical conditions.249–253 In of TCAs for muscarinic cholinergic receptors and typically addition, perhaps due to their ability to inhibit the re-uptake include dry mouth, blurred vision, constipation, urinary hesi- of both serotonin and norepinephrine, as well as their ability tancy, tachycardia, memory difficulties, and ejaculatory diffi- to block sodium channels, TCAs appear to be more effective in culties. Finally, due to their anticholinergic effects, TCAs treating neuropathic pain than the SSRIs.254 In fact, the results should be avoided in patients with narrow-angle glaucoma of a separate meta-analysis reveal that the TCAs are superior to and prostatic hypertrophy, as symptoms related to these con- the SSRIs in the treatment of a number of somatic/pain disor- ditionsy ma worsen because of such anticholinergic effects. ders (including headaches, fibromyalgia, irritable bowel disor- Antihistaminergic-related side effects result from histamin- der, idiopathic pain, tinnitus, and chronic fatigue) often ergic H1-receptor blockade and typically include increased diagnosed in patients with chronic depression.255 The TCAs appetite, weight gain, sedation, and fatigue. Weight gain with may be sub-divided into tertiary amines and secondary amines TCAs can be substantial, averaging 1 to 3 lb per month of (their demethylated secondary amine derivatives). In addition, treatment.260 As a result, TCAs may complicate the manage- maprotiline (Ludiomil), which is classified as a tetracyclic anti- ment of diabetes and worsen glycemic control, and should be depressant, is commonly grouped with the TCAs, due to simi- avoided whenever possible in diabetics.261 yTCAs ma also have larities in dosing, mechanism of action, and side effects. Tertiary hyperlipidemic effects, thus complicating their long-term use amine TCAs include amitriptyline (Elavil, Adepril), imipramine in patients with hyperlipidemia.262 Xerostomia secondary to (Tofranil, Antidepril), trimipramine (Surmontil, Herphonal), anticholinergic and antihistaminergic effects may also increase clomipramine (Anafranil, Clopress), and doxepin (Sinequan, the risk of oral pathology, particularly dental caries.263 Deptran). Secondary amine TCAs are nortriptyline (Pamelor, Orthostatic hypotension and reflex tachycardia may result Aventyl), desipramine (Norpramin, Metylyl), protriptyline from alpha1-adrenergic receptor antagonism. Nortriptyline is (Vivactil, Concordin), and amoxapine (Ascendin, Defanyl). generally thought to be less likely to cause orthostatic hypoten- sion than tertiary amine TCAs, such as imipramine264,265; Side-effect Profile however, nortriptyline’s affinity for the alpha1-adrenergic recep- tor, although less than the affinity of most TCAs, is actually In general, the side effects of the TCAs and related cyclic anti- much greater (e.g., by a factor of two) than the affinity of depressants are more difficult for patients to tolerate than are desipramine and protriptyline.50 In addition, homozygosity for the side effects of the newer drugs (Table 43-2) and they prob- 3435T alleles of ABCB1, the multi-drug resistance gene that ably account for higher drop-out rates than are associated with encodes a P-glycoprotein (P-gp) regulating the passage of many the. SSRIs 256 , Thus treatment is typically initiated at lower substances across the blood–brain barrier, appears to be a risk doses (e.g., 10 mg/day for imipramine) in order to minimize factor for occurrence of nortriptyline-induced postural hypo- the risk of adverse events and premature treatment discontinu- tension.266 Antidepressant-induced postural hypotension in ation. The side-effect profile of the TCAs can be sub-categorized the elderly may, in turn, increase the risk of falls and fractures in terms of their relative affinity for a number of monoamine (e.g., hip fractures).267 Although less likely to suffer a fall or receptors and transporters. Overall, secondary amine TCAs fracture, the sedative potential of various TCA antidepressants tend to cause fewer anticholinergic, antihistaminergic, and is also a serious consideration in younger depressed patients as anti-alpha1–related side effects than tertiary amine TCAs. well, as this effect may increase the mortality risk from

TABLE 43-2 Tricyclic Antidepressants and Monoamine Oxidase Inhibitors Side-effect Profile Sedative Orthostatic Usual Adult Daily Category and Drug Anticholinergic Potency Hypotensive Potency Dose (mg/day) Dosage (mg/day) Tricyclic and Related Cyclic Compounds* Amitriptyline High Very high High 150–200 75–300 Amoxapine Low Moderate Moderate 150–200 75–300 Clomipramine High High High 150–200 75–250 Desipramine Low Moderate (lowest of the ) Moderate 150–200 75–300 Doxepin High High Moderate 150–200 75–300 Imipramine Moderate High High 150–200 75–300 Maprotiline Moderate Low Moderate 150–200 75–225 Nortriptyline Moderate Moderate Lowest of the tricyclics 75–100 40–150 Protriptyline Low High Low 30 10–60 Trimipramine High Moderate Moderate 150–200 75–300 Monoamine Oxidase Inhibitors Isocarboxazid — Very low High 30 20–60 Phenelzine Low Very low High 60–75 30–90 Tranylcypromine — Very low High 30 20–90 *All of the tricyclic and related cyclic compounds have well-established cardiac arrhythmogenic potential. (From Rosenbaum JF, Arana GW, Hyman SE, et al., editors: Handbook of psychiatric drug therapy, ed 5, Philadelphia, 2005, Lippincott Williams & Wilkins.)

Downloaded for Rohul Amin ([email protected]) at Uniformed Services Univ of the Health Sciences from ClinicalKey.com by Elsevier on September 21, 2018. For personal use only. No other uses without permission. Copyright ©2018. Elsevier Inc. All rights reserved. 502 PART X Treatment Approaches automobile accidents. In fact, in a recent review,268 sedating remaining TCAs exhibited a curvilinear (inverse “U”–shaped antidepressants (dothiepin, amitriptyline, imipramine, curve) relationship between blood level and clinical response. doxepin, and mianserin) were found to result in driving impair- The optimal ranges for nortriptyline, “total” imipramine ments on a standardized road test comparable to impairments (imipramine plus desipramine), and “total” amitriptyline found in drivers with a blood level of 0.8 mg/ml, (amitriptyline plus nortriptyline) (with their corresponding whereas non-sedating antidepressants (moclobemide, fluoxet- response rates within versus outside the level range) were 58 to ine, paroxe­tine, venlafaxine, and nefazodone) were not found 148 ng/ml (66% versus 26%), 175 to 350 ng/ml (67% versus to adversely affect driving performance. TCAs may also cause 39%), and 93 to 140 ng/ml (50% versus 30%), respectively. sexual dysfunction and excessive sweating. Nortriptyline levels have been the best studied of the anti- The ability of TCAs to inhibit the sodium channel may also depressants. Some researchers believe that such studies reveal result in electrocardiographic changes in susceptible individu- a more complex pattern than with imipramine or desipramine— als (e.g., in post–myocardial infarction patients, as well as in an inverted U-shape correlation with clinical improvement, patients with bifascicular heart block, left bundle branch which is sometimes referred to as a therapeutic window. Clini- block, or a prolonged QT interval), even at therapeutic doses,269 cal improvement correlates with levels of 50 to 150 ng/ml. The and, given that contemporary psychopharmacologists have reason for the poorer response with doses above 150 ng/ml is access to a multitude of alternative treatment options, TCAs not known, but it does not appear to relate to any measurable should be avoided in these patients. Due to the inhibition of toxicity. On the other hand, the number of subjects in well- sodium channels and cholinergic receptors, the TCAs also designed studies that indicate a window is small, so not all carry a risk of seizure. Maprotiline and clomipramine are con- researchers believe there is adequate evidence in favor of a sidered the TCAs with the greatest risk of seizures.270 This therapeutic window. Studies of amitriptyline levels have combined risk of seizure and arrhythmia renders the TCAs as resulted in disagreement about the utility of levels, with linear, the least safe during overdose.271 curvilinear, and lack of relationship reported by different investigators. When used, blood levels should be drawn when Prescribing Tricyclic and Related Cyclic Antidepressants the drug has achieved steady-state levels (at least 5 days after a dosage change in healthy adults; longer in the elderly) and Aside from the electrocardiogram (ECG), no other tests are 10 to 14 hours after the last oral dose. Abrupt discontinuation generally indicated in healthy adults before starting a TCA. symptoms may emerge with TCAs and in part represent TCAs are started at a low dose followed by gradual increases cholinergic rebound, and include GI distress, malaise, chills, until the therapeutic range is achieved. Finding the right TCA coryza, and muscle aches. dose for a patient often involves a process of trial and error. The most common error leading to treatment failure is inadequate dosage. In healthy adults, the typical starting dose is 25 to Use of Tricyclic and Related Cyclic Antidepressants 50 mg of imipramine or its TCA equivalent. Nortriptyline is during Pregnancy and the Post-partum Period about twice as potent; thus, its starting dose is 10 to 25 mg. In There are limited data on the use of TCAs during pregnancy. some clinical situations, especially in the elderly and patients There have been reports of congenital malformations in asso- with panic disorder, it may be necessary to start with lower ciation with TCA use, but there is no convincing causal assoc­ doses (as low as 10 mg of imipramine or the equivalent) iation. Overall, the TCAs may be safe, but given the lack of because of intolerance to side effects. Generally, TCAs are proven safety, the drugs should be avoided during pregnancy, administered once a day at bedtime to help with compliance unless the indications are compelling. Pregnant women who and, when the sedating compounds are used, to help with are at risk for serious depression might be maintained on TCA sleep. Divided doses are used if patients have side effects due therapy. This decision should always be made very carefully to high peak levels. The dosage can be increased by 50 mg every and with extensive discussion of the risk–benefit factors. Due 3 to 4 days, as side effects allow, up to a dose of 150 to 200 mg to more clinical experience, older agents, such as imipramine, of imipramine or its equivalent at bedtime (see Table 43-1). If may be preferred to newer drugs during pregnancy. TCAs there is no therapeutic response in 3 to 4 weeks, the dosage appear to be secreted in breast milk. Because their effects on should be slowly increased, again as side effects allow. The normal growth and development are unknown, breast-feeding maximum dosage of most TCAs is the equivalent of 300 mg/ should be discouraged for mothers who are on tricyclics. day of imipramine, although uncommonly, patients who metabolize the drug rapidly may do well on higher dosages. Of the currently available cyclic antidepressants, only four drugs Overdoses with Tricyclic and (imipramine, desipramine, amitriptyline, and nortriptyline) Related Cyclic Antidepressants have been studied well enough to make generalizations about the value of their blood levels in treatment of depression. Acute doses of more than 1 g of TCAs are often toxic and may Serum levels of the other cyclic antidepressants have not been be fatal. Death may result from cardiac arrhythmias, hypoten- investigated well enough to be clinically meaningful, although sion, or uncontrollable seizures. Serum levels should be they can confirm presence of the drug or document extremely obtained when overdose is suspected, both because of dis- high serum levels. There is a wide range of effective doses for torted information that may be given by patients or families TCAs. Typical antidepressant doses are 100 to 300 mg/day for and because oral with very large doses of these imipramine. There is evidence to suggest a relationship between compounds is poorly understood. Nonetheless, serum levels serum levels of TCAs and clinical response. Perry and col- of the parent compound and its active metabolites provide leagues272 pooled and analyzed all available studies examining less specific information about the severity of the overdose the relationship between TCA blood levels and clinical response than one might hope. Serum levels of greater than 1,000 ng/ with the use of receiver operating-characteristics curves. The ml are associated with serious overdose, as are increases in the relationship between clinical response and blood levels for QRS duration of the ECG to 0.10 second or greater. However, desipramine was linear, with the threshold concentration in serious consequences of a TCA overdose may occur with serum plasma for therapeutic response being greater than or equal levels under 1,000 ng/ml and with a QRS duration of less than to 116 ng/ml (response rates: 51% versus 15% for patients 0.10 second. In acute overdose, almost all symptoms develop with levels above or below that threshold, respectively). The within 12 hours.

Downloaded for Rohul Amin ([email protected]) at Uniformed Services Univ of the Health Sciences from ClinicalKey.com by Elsevier on September 21, 2018. For personal use only. No other uses without permission. Copyright ©2018. Elsevier Inc. All rights reserved. Antidepressants 503

metabolites. There is controversy as to whether phenelzine is BOX 43-3 Drug Interactions with Cyclic Antidepressants cleaved and acetylated in the liver. It is known that a sizable 43 number of people are slow acetylators (a high percentage of WORSEN SEDATION Asians and about 50% of whites and blacks), but there is little Alcohol evidence that the rate of acetylation is clinically significant for this class of drugs. Of clinical importance is the observation that metabolism of MAOIs does not seem to be affected by , chloral hydrate, and other sedatives use of anticonvulsants. WORSEN HYPOTENSION Although the overall efficacy of MAOIs does not differ from α- (Aldomet) that of other commonly used antidepressants in the treatment β-Adrenergic blockers (e.g., ) of MDD, their use is considerably limited by the risk of poten- tially lethal adverse events, such as hypertensive crises and Diuretics serotonin syndromes, and by the strict dietary restrictions Low-potency antipsychotics required to minimize such risks. As a result, they are rarely chosen as first-line agents in the treatment of depression84; ADDITIVE CARDIOTOXICITY their use is mainly limited to the treatment of treatment- Quinidine and other type 1 antiarrhythmics refractory depression, either as a “next-step” strategy in TCA- , , , resistant depression,273–279 or even depression resistant to a ADDITIVE ANTICHOLINERGIC TOXICITY number of antidepressants.280–284 High doses of the MAOI tra- Anti-histamines ( and others) nylcypromine (90 to 170 mg daily) may also be effective in Anti-parkinsonians (benztropine and others) depressed patients who do not experience sufficient improve- 285 Low-potency antipsychotics, especially thioridazine ment during treatment with lower doses. In addition, Over-the-counter sleeping medications perhaps due to their ability to inhibit the re-uptake of Gastrointestinal antispasmodics and anti-diarrheals (Lomotil and dopamine in addition to serotonin and norepinephrine, 286 others) MAOIs appear to be more effective than TCAs in the treat- ment of atypical depression (characterized by mood reactivity OTHER in addition to symptoms such as hypersomnia, hyperphagia, Tricyclics may increase the effects of warfarin extreme fatigue, and rejection sensitivity). In parallel, while Tricyclics may block the effects of the MAOIs also seem to be effective in the treatment of fatigue in fibromyalgia or chronic fatigue syndrome,287–291 efour of fiv (From Rosenbaum JF, Arana GW, Hyman SE, et al., editors: Handbook 292–296 of psychiatric drug therapy, ed 5, Philadelphia, 2005, Lippincott studies do not show any effect of the SSRIs on fatigue. Williams & Wilkins.) Although, to date, there are no double-blind studies that compare the relative efficacy of MAOIs versus the SSRIs or TCAs in the treatment of fatigue in depression, these studies suggest a potential advantage for MAOIs over SSRIs. In the GI tract and the liver, MAO catabolizes a number of dietary pressor amines (such as dopamine, tyramine, tryp- tamine, and phenylethylamine).52 For this reason, consump- Anti-muscarinic effects are prominent, including dry tion of foods containing high levels of dietary amines while mucous membranes, warm dry skin, mydriasis, blurred vision, onOI an MA may precipitate a hypertensive crisis, character- decreased bowel motility, and, often, . Either izedy b hypertension, hyperpyrexia, tachycardia, tremulous- CNS depression (ranging from drowsiness to coma) or an ness, and cardiac arrhythmias.53 The same reaction may also agitated delirium may occur. The CNS depressant effects of occur during co-administration of dopaminergic agents and cyclic antidepressants are potentiated by concomitantly MAOIs, while the co-administration of MAOIs with other anti- ingested alcohol, benzodiazepines, and other sedative- depressants that potentiate serotonin could result in serotonin hypnotics. Seizures may occur, and in severe overdoses, respi- syndromes due to toxic CNS serotonin levels. The serotonin ratory arrest may occur. Cardiovascular toxicity presents a syndrome is characterized by alterations in cognition (disori- particular danger (Box 43-3). Hypotension often occurs, even entation, confusion), behavior (agitation, restlessness), auto- with the patient supine. nomic nervous system function (fever, shivering, diaphoresis, A variety of arrhythmias may develop, including supraven- diarrhea), and neuromuscular activity (ataxia, hyperreflexia, tricular tachycardia, ventricular tachycardia, or fibrillation, and and myoclonus).54–56 OSince MA enzymatic activity requires varying degrees of heart block, including complete heart block. approximately 14 days to be restored, such food or medica- tions should be avoided for 2 weeks after the discontinuation Drug Interactions of an irreversible MAOI (“MAOI washout period”). Seroton- The cyclic antidepressants have a variety of important phar- ergic and dopaminergic antidepressants are typically discon- macodynamic and pharmacokinetic drug–drug interactions tinued 2 weeks before the initiation of an MAOI, with the thaty ma worsen the toxicity of other drugs (see Box 43-3). exception of fluoxetine, which needs to be discontinued 5 weeks in advance due to its relatively longer half-life. In addi- Monoamine Oxidase Inhibitors tion to its oral formulation, selegiline is also available in a transdermal form (patch), designed to minimize the inhibi- The MAOIs are well absorbed after oral administration. A tion of the MAO enzymes found in the lining of the GI tract. transdermal form of selegiline (Emsam) is also available and Treating MDD with transdermal selegiline appears to be effec- approvedy b the FDA for the treatment of depression. Since tive297,298 and also safe, even in the absence of a tyramine- MAOIs irreversibly inhibit the enzymes, return of enzyme restricted diet.298 Although rare, serotonin syndrome may function after discontinuation may require 2 weeks (the time occur when oral selegiline is combined with serotonergic it takes for de novo synthesis of the enzyme). The metabolism agents, particularly the SSRIs.299 The risk of such drug interac- of MAOIs is not well understood. Selegiline and tranylcy- tions with the transdermal formulation of selegiline has not promine have meta-amphetamine and amphetamine as been studied.

Downloaded for Rohul Amin ([email protected]) at Uniformed Services Univ of the Health Sciences from ClinicalKey.com by Elsevier on September 21, 2018. For personal use only. No other uses without permission. Copyright ©2018. Elsevier Inc. All rights reserved. 504 PART X Treatment Approaches

Side-effect Profile syndrome. MAOIs must be used with caution in patients with diabetes (due to possible potentiation of oral hypoglycemics The most common side effects of MAOIs include postural and worsened hypoglycemia). hypotension, insomnia, agitation, sedation, and sexual dys- function, although the incidence of sexual dysfunction is Dosage lower than with the SSRIs.300 Other side effects include weight change, dry mouth, constipation, and urinary hesitancy.51 The optimal dosages for MAOIs vary from agent to agent. Peripheral neuropathies have been reported, and may be pre- Initially, MAOIs are administered at low doses, with gradual ventedy b concomitant therapy with pyridoxine.301 A list of increases as side effects allow. Some tolerance may develop to side effects with MAOIs is reported in Table 43-2. Elderly side, effects including postural hypotension. Phenelzine is patients may develop constipation or urinary retention. usually started at 15 mg twice daily (7.5 to 15 mg/day in the Alternatively, nausea and diarrhea have been reported by elderly), isocarboxazid at 10 mg twice daily, and tranylcy- some patients. Sweating, flushing, or chills may occur. Rarely, promine at 10 mg twice daily (5 to 10 mg/day in the elderly). hepatotoxicity (which may be serious) may occur with Dosages can be increased by 15 mg weekly for phenelzine and phenelzine. Peripheral edema likely reflecting effects of the 10 mg weekly for isocarboxazid and tranylcypromine (as side drug on small vessels may prove difficult to manage. Finally, effects allow) to 45 to 60 mg/day for phenelzine (30 to 60 mg/ some patients complain of muscle twitching or electric day in the elderly) and 30 to 40 mg/day for the others. Dosages shock–like sensations. as0 high as 9 mg/day of these drugs may be required, although these exceed the manufacturer’s recommendations. Once Dietary Restrictions and Drug Interactions depressive symptoms remit, full therapeutic doses are protec- tive against relapse, although in managing patients on MAOIs, As discussed previously, treatment with MAOIs carries a risk of dose adjustments over time to manage side effects or clinical hypertensive crisis. To minimize this risk, patients on MAOIs response are common. For transdermal selegiline, the minimal need to adhere to a strict dietary regimen that excludes foods effective dose reported is 6 mg/day. It is prudent to taper and beverages that have a high content of dietary amines, MAOIs over 2 weeks or more when discontinuing them including all aged cheeses; sour cream; yogurt; fermented or because discontinuation reactions have been reported with dried meats (sausages, basderma, pastrami, pepperoni, louza, abrupt discontinuation. There is little experience with the use lingiça, chorizo); offal (liver, sweetbread, , tripe, brains); of MAOIs in pregnancy. For this reason, their use should fava and broad bean pods (lima, lentils, snow-peas); marmite be avoided. yeast extract; sauerkraut; soy sauce and other soy products; over- Patients have reported weight gain on all MAOIs and occa- ripe bananas and avocado; eggplant; spinach; pickled, dried, or sionally weight loss (more commonly on tranylcypromine). salted fish; caviar; fish roe (tarama); and foods containing Anticholinergic-like side effects occur, although they are not monosodium glutamate (MSG). Patients should also avoid due to muscarinic antagonism. These side effects are less severe consumption of caffeinated drinks, and most alcoholic bever- than those seen with TCAs, although patients on phenelzine ages, especially tap beer and red wine, but also certain white may experience dry mouth. Elderly patients may develop con- wines, including those that are resinated (retsina), botrytized stipation or urinary retention. Nausea and diarrhea have been (sauternes, cadillac, loupiac, monbazillac, coteaux du layon, reportedy b some patients. Sweating, flushing, or chills also Alsace vendage tardive, tokaji aszú, trockenbeerenausle), aged may. occur Rarely, hepatotoxicity may occur with phenelzine, (sherry), and others (Riesling, vermouth). Sympathomimetics, whichy ma be serious. Peripheral edema likely reflecting effects both prescribed and over-the-counter (, ephe- of the drug on small vessels may prove difficult to manage. drine, , , and methylpheni- Finally, some patients complain of muscle twitching or electric date), potent noradrenergic and dopaminergic antidepressants, shock–like sensations. The latter may respond to , , and meperidine (Demerol) may also pre- although the emergence of neurological or neuropathic symp- cipitate a hypertensive crisis. In addition, as mentioned previ- tomsy ma reflect interference with absorption of vitamin B6 ously, combining MAOIs with potent serotonergic agents (such that should improve with dietary supplementation of pyridox- as the TCAs, SSRIs, and others) carries a risk of serotonergic ine (vitamin B6)0 5 to 100 mg/day.

TABLE 43-3 Interactions of Monoamine Oxidase Inhibitors with Other Drugs* Drug Effect Sympathomimetics (e.g., , dopamine, , Hypertensive crisis epinephrine [], isoproterenol [Isuprel], , , oxymetazoline [Afrin], norepinephrine, [Neo-], , pseudoephedrine [Sudafed]) Meperidine (Demerol and others) Fever, delirium, hypertension, hypotension, neuromuscular excitability, death Oral hypoglycemics Further lowering of serum glucose L-dopa Hypertensive crisis Tricyclic antidepressants,† duloxetine, venlafaxine, SSRIs, Fever, seizures, delirium clomipramine, tryptophan Nausea, confusion, anxiety, shivering, hyperthermia, rigidity, diaphoresis, hyperreflexia, tachycardia, hypotension, coma, death Bupropion Hypertensive crisis *This may include selegiline even at low doses. †Tricyclics and MAOIs are occasionally used together. SSRIs, Selective serotonin re-uptake inhibitors. (From Rosenbaum JF, Arana GW, Hyman SE, et al., editors: Handbook of psychiatric drug therapy, ed 5, Philadelphia, 2005, Lippincott Williams & Wilkins.)

Downloaded for Rohul Amin ([email protected]) at Uniformed Services Univ of the Health Sciences from ClinicalKey.com by Elsevier on September 21, 2018. For personal use only. No other uses without permission. Copyright ©2018. Elsevier Inc. All rights reserved. Antidepressants 505

Overdose 103. Fava M, Judge R, Hoog SL, et al. Fluoxetine versus sertraline and paroxetine in major depressive disorder: changes in weight 43 MAOIs are extremely dangerous in overdose. Because they with long-term treatment. J Clin Psychiatry 61(11):863–867, circulate at very low concentrations in serum and are difficult 2000. to assay, there are no good data on therapeutic or toxic serum 106. Thase ME, Ferguson JM, Lydiard RB, et al. Citalopram treatment levels. Manifestations of toxicity may appear slowly, often of paroxetine-intolerant depressed patients. Depress Anxiety 16(3):128–133, 2002. taking up to 12 hours to appear and 24 hours to reach their 107. Thase ME, Blomgren SL, Birkett MA, et al. Fluoxetine treatment peak;, thus even if patients appear clinically well in the emer- of patients with major depressive disorder who failed initial gency department, they should be admitted for observation treatment with sertraline. J Clin Psychiatry 58(1):16–21, 1997. aftery an significant overdose. After an asymptomatic period, 116. Fava M. Prospective studies of adverse events related to antide- a serotonin syndrome may occur, including hyperpyrexia and pressant discontinuation. J Clin Psychiatry 67(Suppl. 4):14–21, autonomic excitation. Neuromuscular excitability may be 2006. severe enough to produce rhabdomyolysis, which may cause 125. Thase ME, Entsuah AR, Rudolph RL. Remission rates during renal failure. This phase of excitation may be followed by CNS treatment with venlafaxine or selective serotonin reuptake inhib- depression and cardiovascular collapse. Death may occur early itors. Br J Psychiatry 178:234, 2001. 140. Clayton AH, Pradko JF, Croft HA, et al. Prevalence of sexual due to seizures or arrhythmias, or later due to asystole, arrhyth- dysfunction among newer antidepressants. J Clin Psychiatry mias, hypotension, or renal failure. Hemolysis and a coagu- 63(4):357–366, 2002. lopathy also may occur and contribute to morbidity and 144. Thase ME. Effects of venlafaxine on blood pressure: a meta- mortality risk. analysis of original data from 3744 depressed patients. J Clin Psychiatry 59(10):502–508, 1998. Drug Interactions 145. Fava M, Mulroy R, Alpert J, et al. Emergence of adverse events following discontinuation of treatment with extended-release Important drug interactions with MAOIs are listed in venlafaxine. Am J Psychiatry 154(12):1760–1762, 1997. Table 43-3. 157. Papakostas GI, Fava M. A meta-analysis of clinical trials compar- ing milnacipran, a serotonin-norepinephrine , Access the complete reference list and multiple choice questions with a selective serotonin reuptake inhibitor for the treatment (MCQs) online at https://expertconsult.inkling.com of major depressive disorder. Eur Neuropsychopharmacol 17(1): 32–36, 2007. 178. Papakostas GI. Dopaminergic-based pharmacotherapies for depression. Eur Neuropsychopharmacol 16(6):391–402, 2006. KEY REFERENCES 179. Trivedi MH, Rush AJ, Carmody TJ, et al. Do bupropion SR and 5. Fava M, Kendler KS. Major depressive disorder. Neuron sertraline differ in their effects on anxiety in depressed patients? 28(2):335–341, 2000. J Clin Psychiatry 62(10):776–781, 2001. 51. Fava M, Rosenbaum JF. Pharmacotherapy and somatic therapies. 180. Papakostas GI, Nutt DJ, Hallett LA, et al. Resolution of sleepi- In Beckham EE, Leber WR, editors: Handbook of depression, New ness and fatigue in major depressive disorder: a comparison of York, 1995, Guilford. bupropion and the selective serotonin reuptake inhibitors. Biol 76. Thase ME. Preventing relapse and recurrence of depression: Psychiatry 60(12):1350–1355, 2006. a brief review of therapeutic options. CNS Spectr 11(12 Suppl. 193. Papakostas GI, Fava M. A meta-analysis of clinical trials compar- 15):12–21, 2006. ing the serotonin (5HT)-2 receptor antagonists trazodone and 78. Papakostas GI, Perlis RH, Seifert C, et al. Antidepressant dose- nefazodone with a selective serotonin reuptake inhibitor for the reduction and the risk of relapse in major depressive disorder. treatment of major depressive disorder. Eur Psychiatry 22(7):444– Psychother Psychosom 76(5):266–270, 2007. 447, 2007. 79. Fava M, Detke MJ, Balestrieri M. Management of depression 230. Papakostas GI, Homberger CH, Fava M. A meta-analysis of clini- relapse: re-initiation of duloxetine treatment or dose increase. cal trials comparing mirtazapine with a selective serotonin J Psychiatr Res 40(4):328–336, 2006. reuptake inhibitor for the treatment of major depressive disor- 80. Papakostas GI, Fava M. A meta-analysis of clinical trials compar- der. J Psychopharmacol 22(8):843–848, 2008. ing moclobemide with selective serotonin reuptake inhibitors 237. Schatzberg AF, Kremer C, Rodrigues HE, et al. Mirtazapine vs. for the treatment of major depressive disorder. Can J Psychiatry Paroxetine Study Group: Double-blind, randomized compari- 51(12):783–790, 2006. son of mirtazapine and paroxetine in elderly depressed patients. 82. Fava M, Hoog SL, Judge RA, et al. Acute efficacy of fluoxetine Am J Geriatr Psychiatry 10(5):541–550, 2002. versus sertraline and paroxetine in major depressive disorder 260. Fava M. Weight gain and antidepressants. J Clin Psychiatry including effects of baseline insomnia. J Clin Psychopharmacol 61(Suppl. 11):37–41, 2000. 22(2):137–147, 2002. 274. McGrath PJ, Stewart JW, Nunes EV, et al. A double-blind cross­ 85. Fava M, Uebelacker LA, Alpert JE, et al. Major depressive sub- over trial of imipramine and phenelzine for outpatients with types and treatment response. Biol Psychiatry 42(7):568–576, treatment-refractory depression. Am J Psychiatry 150(1):118–123, 1997. 1993. 90. Fava M, Rankin M. Sexual functioning and SSRIs. J Clin Psychiatry 279. McGrath PJ, Stewart JW, Harrison W, et al. Treatment of tricyclic 63(Suppl. 5):13–16, discussion 23–25, 2002. refractory depression with a monoamine oxidase inhibitor anti- 92. Fava M, Graves LM, Benazzi F, et al. A cross-sectional study of depressant. Psychopharmacol Bull 23(1):169–172, 1987. the prevalence of cognitive and physical symptoms during long- 286. Thase ME, Trivedi MH, Rush AJ. MAOIs in the contemporary term antidepressant treatment. J Clin Psychiatry 67(11):1754– treatment of depression. Neuropsychopharmacology 12(3):185– 1759, 2006. 219, 1995.

Downloaded for Rohul Amin ([email protected]) at Uniformed Services Univ of the Health Sciences from ClinicalKey.com by Elsevier on September 21, 2018. For personal use only. No other uses without permission. Copyright ©2018. Elsevier Inc. All rights reserved. Antidepressants 505.e1

and serotonergic mechanisms. Behav Brain Res 114(1–2):51–56, REFERENCES 2000. 43 1. Crane GE. The psychiatric side-effects of iproniazid. Am J Psy- 24. Marcoli M, Maura G, Tortarolo M, et al. Trazodone is a potent chiatry 112:494–501, 1956. agonist at 5-HT2C receptors mediating inhibition of the 2. Kuhn R. The treatment of depressive states with G22355 (imi- N-methyl-D-aspartate//cyclic GMP pathway in rat pramine hydrochloride). Am J Psychiatry 115:459–464, 1958. cerebellum. J Pharmacol Exp Ther 285(3):983–986, 1998. 3. Gershon S, Homberg G, Mattson E, et al. Imipramine hydro- 25. Maura G, Marcoli M, Pepicelli O, et al. Serotonin inhibition of chloride: its effects on clinical, autonomic, and psychological the NMDA receptor/nitric oxide/cyclic GMP pathway in human functions. Arch Gen Psychiatry 6:96–101, 1962. neocortex slices: involvement of 5-HT(2C) and 5-HT(1A) recep- 4. Glowinski J, Axelrod J. Inhibition of uptake of tritiated- tors. Br J Pharmacol 130(8):1853–1858, 2000. noradrenaline in the intact rat brain by imipramine and structur- 26. Pazzagli M, Giovannini MG, Pepeu G. Trazodone increases extra- ally related compounds. Nature 204:1318–1319, 1964. cellular serotonin levels in the frontal cortex of rats. Eur J Phar- 5. Fava M, Kendler KS. Major depressive disorder. Neuron 28(2): macol 383(3):249–257, 1999. 335–341, 2000. 27. Rao R. Serotonergic syndrome with trazodone. Hosp Med 6. Hyman SE, Nestler EJ. Initiation and adaptation: a paradigm for 59(1):79, 1998. understanding psychotropic drug action. Am J Psychiatry 28. Millan MJ, Gobert A, Lejeune F, et al. The novel melatonin 153(2):115–162, 1996. agonist agomelatine (S20098) is an antagonist at 7. Kalia M. Neurobiological basis of depression: an update. Metabo- 5-hydroxytryptamine 2C receptors, blockade of which enhances lism 54(5 Suppl. 1):24–27, 2005. the activity of frontocortical dopaminergic and adrenergic path- 8. Chen H, Pandey GN, Dwivedi Y. Hippocampal cell proliferation ways. J Pharmacol Exp Ther 306(3):954–964, 2003. regulation by repeated stress and antidepressants. Neuroreport 29. Blier P, Ward NM. Is there a role for 5-HT1A agonists in the 17(9):863–867, 2006. treatment of depression? Biol Psychiatry 53(3):193–203, 2003. 9. Rantamaki T, Hendolin P, Kankaanpaa A, et al. Pharmacologi- 30. Rimele TJ, Henry DE, Lee DK, et al. Tissue-dependent alpha cally diverse antidepressants rapidly activate brain-derived neu- adrenoceptor activity of buspirone and related compounds. rotrophic factor receptor TrkB and induce phospholipase-c J Pharmacol Exp Ther 241(3):771–778, 1987. gamma signaling pathways in mouse brain. Neuropsychopharma- 31. Bergman J, Roof RA, Furman CA, et al. Modification of cology 32(10):2152–2162, 2007. self-administration by buspirone (buspar®): potential involve- 10. Richelson E. Interactions of antidepressants with neurotransmit- ment of D3 and D4 dopamine receptors. Int J Neuropsychophar- ter transporters and receptors and their clinical relevance. J Clin macol 16(2):445–458, 2013. Psychiatry 64(Suppl. 13):5–12, 2003. 32. Piercey MF, Smith MW, Lum-Ragan JT. Excitation of noradrener- 11. Barbon A, Popoli M, La Via L, et al. Regulation of editing and gic cell firing by 5-hydroxytryptamine 1A agonists correlates with expression of glutamate alpha-amino-propionic-acid (AMPA)/ dopamine antagonist properties. J Pharmacol Exp Ther 268(3): kainate receptors by antidepressant drugs. Biol Psychiatry 1297–1303, 1994. 59(8):713–720, 2006. 33. Lechin F, van der Dijs B, Jara H, et al. Effects of buspirone on 12. Xu H, Chen Z, He J, et al. Synergetic effects of and plasma neurotransmitters in healthy subjects. J Neural Transm venlafaxine in preventing the chronic restraint stress-induced 105(6–7):561–573, 1998. decrease in cell proliferation and BDNF expression in rat hip- 34. Astier B, Lambas Senas L, Souliere F, et al. In vivo comparison pocampus. Hippocampus 16(6):551–559, 2006. of two 5-HT1A receptors agonists (S-20499) and 13. Calabrese F, Molteni R, Maj PF, et al. Chronic duloxetine treat- buspirone on locus coeruleus neuronal activity. Eur J Pharmacol ment induces specific changes in the expression of BDNF tran- 459(1):17–26, 2003. scripts and in the subcellular localization of the neurotrophin 35. Gobert A, Rivet JM, Cistarelli L, et al. Buspirone modulates basal protein. Neuropsychopharmacology 32(11):2351–2359, 2007. and fluoxetine-stimulated dialysate levels of dopamine, 14. Bymaster FP, Katner JS, Nelson DL, et al. Atomoxetine increases noradrenaline and serotonin in the frontal cortex of freely extracellular levels of norepinephrine and dopamine in prefron- moving rats: activation of serotonin1A receptors and blockade tal cortex of rat: a potential mechanism for efficacy in attention of alpha2-adrenergic receptors underlie its actions. Neuroscience deficit/hyperactivity disorder. Neuropsychopharmacology 27(5): 93(4):1251–1262, 1999. 699–711, 2002. 36. van Amsterdam C, Seyfried CA. Mechanism of action of the 15. Miller DK, Wong EH, Chesnut MD, et al. Reboxetine: functional bimodal antidepressant vilazodone: evidence for serotonin1A- inhibition of monoamine transporters and nicotinic acetylcho- receptor-mediated auto-augmentation of extracellular serotonin line receptors. J Pharmacol Exp Ther 302(2):687–695, 2002. output. Psychopharmacology (Berl) 231(12):2547–2558, 2014. 16. Tzavara ET, Bymaster FP, Overshiner CD, et al. Procholinergic 37. Guilloux JP, Mendez-David I, Pehrson A, et al. Antidepressant and memory enhancing properties of the selective norepine- and potential of the multimodal antidepressant vorti- phrine uptake inhibitor atomoxetine. Mol Psychiatry 11(2):187– oxetine (Lu AA21004) assessed by behavioural and neurogenesis 195, 2006. outcomes in mice. Neuropharmacology 73:147–159, 2013. 17. Creighton CJ, Ramabadran K, Ciccone PE, et al. Synthesis and 38. Mørk A, Pehrson A, Brennum LT, et al. Pharmacological effects biological evaluation of the major metabolite of atomoxetine: of Lu AA21004: a novel multimodal compound for the treatment elucidation of a partial kappa-opioid agonist effect. Bioorg Med of major depressive disorder. J Pharmacol Exp Ther 340(3):666– Chem Lett 14(15):4083–4085, 2004. 675, 2010. 18. Haria M, Fitton A, McTavish D. Trazodone. A review of its phar- 39. Ascher JA, Cole JO, Colin JN, et al. Bupropion: a review of its macology, therapeutic use in depression and therapeutic poten- mechanism of antidepressant activity. J Clin Psychiatry 56(9):395– tial in other disorders. Drugs Aging 4(4):331–355, 1994. 401, 1995. 19. Taylor DP, Carter RB, Eison AS, et al. Pharmacology and neuro- 40. Learned-Coughlin SM, Bergstrom M, Savitcheva I, et al. In vivo chemistry of nefazodone, a novel antidepressant drug. J Clin activity of bupropion at the human dopamine transporter as Psychiatry 56(Suppl. 6):3–11, 1995. measured by positron emission tomography. Biol Psychiatry 20. Meyer JH, Cho R, Kennedy S, et al. The effects of single dose 54(8):800–805, 2003. nefazodone and paroxetine upon 5-HT2A binding potential in 41. Foley KF, Cozzi NV. Inhibition of transport function and humans using [18F]- PET. Psychopharmacology (Berl) desipramine binding at the human noradrenaline transporter by 144(3):279–281, 1999. N-ethylmaleimide and protection by substrate analogs. Naunyn 21. Hemrick-Luecke SK, Snoddy HD, Fuller RW. Evaluation of Schmiedebergs Arch Pharmacol 365(6):457–461, 2002. nefazodone as a serotonin uptake inhibitor and a serotonin 42. Dong J, Blier P. Modification of norepinephrine and serotonin, antagonist in vivo. Life Sci 55(7):479–483, 1994. but not dopamine, neuron firing by sustained bupropion treat- 22. Rothman RB, Baumann MH. Serotonin releasing agents. Neuro- ment. Psychopharmacology (Berl) 155(1):52–57, 2001. chemical, therapeutic and adverse effects. Pharmacol Biochem 43. Li SX, Perry KW, Wong DT. Influence of fluoxetine on the ability Behav 71(4):825–836, 2002. of bupropion to modulate extracellular dopamine and nor­ 23. Schreiber S, Backer MM, Herman I, et al. The antinociceptive epinephrine concentrations in three mesocorticolimbic areas of effect of trazodone in mice is mediated through both mu-opioid rats. Neuropharmacology 42(2):181–190, 2002.

Downloaded for Rohul Amin ([email protected]) at Uniformed Services Univ of the Health Sciences from ClinicalKey.com by Elsevier on September 21, 2018. For personal use only. No other uses without permission. Copyright ©2018. Elsevier Inc. All rights reserved. 505.e2 PART X Treatment Approaches

44. Miller DK, Sumithran SP, Dwoskin LP. Bupropion inhibits 65. Ebert D, Albert R, May A, et al. Combined SSRI-RIMA treatment -evoked [(3)H]overflow from rat striatal slices preloaded in refractory depression. Safety data and efficacy. Psychopharma- with [(3)H]dopamine and from rat hippocampal slices preloaded cology (Berl) 119(3):342–344, 1995. with [(3)H]norepinephrine. J Pharmacol Exp Ther 302(3):1113– 66. Joffe RT, Bakish D. Combined SSRI-moclobemide treatment of 1122, 2002. psychiatric illness. J Clin Psychiatry 55(1):24–25, 1994. 45. Haddjeri N, Blier P, de Montigny C. Noradrenergic modulation 67. Chambost M, Liron L, Peillon D, et al. Serotonin syndrome of central serotonergic neurotransmission: acute and long-term during fluoxetine poisoning in a patient taking moclobemide. actions of mirtazapine. Int Clin Psychopharmacol 10(Suppl. 4):11– Can J Anaesth 47(3):246–250, 2000. 17, 1995. 68. Dardennes RM, Even C, Ballon N, et al. Serotonin syndrome 46. Anttila SA, Leinonen EV. A review of the pharmacological and caused by a clomipramine-moclobemide interaction. J Clin Psy- clinical profile of mirtazapine. CNS Drug Rev 7(3):249–264, chiatry 59(7):382–383, 1998. 2001. 69. Hojer J, Personne M, Skagius AS, et al. Serotonin syndrome— 47. Stoll L, Seguin S, Gentile L. Tricyclic antidepressants, but not the several cases of this often overlooked diagnosis. Tidsskr Nor selective serotonin reuptake inhibitor fluoxetine, bind to the Laegeforen 122(17):1660–1663, 2002. S1S2 domain of AMPA receptors. Arch Biochem Biophys 458(2): 70. Isbister GK, McGettigan P, Dawson A. A fatal case of moclobemide- 213–219, 2007. citalopram intoxication. J Anal Toxicol 25(8):716–717, 2001. 48. Nunez E, Lopez-Corcuera B, Vazquez J, et al. Differential effects 71. Dams R, Benijts TH, Lambert WE, et al. A fatal case of serotonin of the amoxapine on glycine uptake syndrome after combined moclobemide-citalopram intoxica- mediated by the recombinant GLYT1 and GLYT2 glycine trans- tion. J Anal Toxicol 25(2):147–151, 2001. porters. Br J Pharmacol 129(1):200–206, 2000. 72. Rogde S, Hilberg T, Teige B. Fatal combined intoxication with 49. Kapur S, Cho R, Jones C, et al. Is amoxapine an atypical anti­ new antidepressants. Human cases and an experimental study psychotic? Positron-emission tomography investigation of its of postmortem moclobemide redistribution. Forensic Sci Int dopamine2 and serotonin2 occupancy. Biol Psychiatry 45(9):1217– 100(1–2):109–116, 1999. 1220, 1999. 73. Finge T, Malavialle C, Lambert J. Fatal form of serotonin syn- 50. Richelson E, Nelson A. Antagonism by antidepressants of neu- drome. Ann Fr Anesth Reanim 16(1):80–81, 1997. rotransmitter receptors of normal human brain in vitro. J Phar- 74. Singer PP, Jones GR. An uncommon fatality due to moclobemide macol Exp Ther 230:94–102, 1984. and paroxetine. J Anal Toxicol 21(6):518–520, 1997. 51. Fava M, Rosenbaum JF. Pharmacotherapy and somatic therapies. 75. Geddes JR, Carney SM, Davies C, et al. Relapse prevention with In Beckham EE, Leber WR, editors: Handbook of depression, New antidepressant drug treatment in depressive disorders: a system- York, 1995, Guilford. atic review. Lancet 361(9358):653–661, 2003. 52. Hasan F, McCrodden JM, Kennedy NP, et al. The involvement of 76. Thase ME. Preventing relapse and recurrence of depression: a intestinal monoamine oxidase in the transport and metabolism brief review of therapeutic options. CNS Spectr 11(12 Suppl. of tyramine. J Neural Transm Suppl 26:1–9, 1988. 15):12–21, 2006. 53. Lavin MR, Mendelowitz A, Kronig MH. Spontaneous hyperten- 77. Kupfer DJ, Frank E, Perel JM, et al. Five-year outcome for main- sive reactions with monoamine oxidase inhibitors. Biol Psychiatry tenance therapies in recurrent depression. Arch Gen Psychiatry 34(3):146–151, 1993. 49(10):769–773, 1992. 54. Lane R, Baldwin D. Selective serotonin reuptake inhibitor– 78. Papakostas GI, Perlis RH, Seifert C, et al. Antidepressant dose- induced serotonin syndrome: review. J Clin Psychopharmacol reduction and the risk of relapse in major depressive disorder. 17(3):208–221, 1997. Psychother Psychosom 76(5):266–270, 2007. 55. Feighner JP, Boyer WF, Tyler DL, et al. Adverse consequences of 79. Fava M, Detke MJ, Balestrieri M. Management of depression fluoxetine-MAOI combination therapy. J Clin Psychiatry 51(6): relapse: re-initiation of duloxetine treatment or dose increase. 222–225, 1990. J Psychiatr Res 40(4):328–336, 2006. 56. Beasley CM Jr, Masica DN, Heiligenstein JH, et al. Possible 80. Papakostas GI, Fava M. A meta-analysis of clinical trials compar- monoamine oxidase inhibitor–serotonin uptake inhibitor inter- ing moclobemide with selective serotonin reuptake inhibitors action: fluoxetine clinical data and preclinical findings. J Clin for the treatment of major depressive disorder. Can J Psychiatry Psychopharmacol 13(5):312–320, 1993. 51(12):783–790, 2006. 57. Bonnet U. Moclobemide: therapeutic use and clinical studies. 81. Pande AC, Birkett M, Fechner-Bates S, et al. Fluoxetine versus CNS Drug Rev 9(1):97–140, 2003. phenelzine in atypical depression. Biol Psychiatry 40(10):1017– 58. Levant B, Moehlenkamp JD, Morgan KA, et al. Modulation of 1020, 1996. [3H]quinpirole binding in brain by monoamine oxidase inhibi- 82. Fava M, Hoog SL, Judge RA, et al. Acute efficacy of fluoxetine tors: evidence for a potential novel binding site. J Pharmacol Exp versus sertraline and paroxetine in major depressive disorder Ther 278(1):145–153, 1996. including effects of baseline insomnia. J Clin Psychopharmacol 59. Levant B, Bancroft GN. Inhibition of [3H]quinpirole binding by 22(2):137–147, 2002. a monoamine oxidase inhibitor in subcellular fractions of rat 83. Kennedy SH, Andersen HF, Lam RW. Efficacy of escitalopram in striatum. Life Sci 63(18):1643–1651, 1998. the treatment of major depressive disorder compared with con- 60. Slawson MH, Taccogno JL, Foltz RL, et al. Quantitative analysis ventional selective serotonin reuptake inhibitors and venlafaxine of selegiline and three metabolites (N-desmethylselegiline, XR: a meta-analysis. J Psychiatry Neurosci 31(2):122–131, 2006. , and amphetamine) in human plasma by 84. Petersen T, Dording C, Neault NB, et al. A survey of prescribing high-performance liquid chromatography–atmospheric pressure practices in the treatment of depression. Prog Neuropsychophar- chemical ionization–tandem mass spectrometry. J Anal Toxicol macol Biol Psychiatry 26(1):177–187, 2002. 26(7):430–437, 2002. 85. Fava M, Uebelacker LA, Alpert JE, et al. Major depressive sub- 61. Baker GB, Urichuk LJ, McKenna KF, et al. Metabolism of types and treatment response. Biol Psychiatry 42(7):568–576, monoamine oxidase inhibitors. Cell Mol Neurobiol 19(3):411– 1997. 426, 1999. 86. Papakostas GI, Petersen T, Iosifescu DV, et al. Somatic symptoms 62. Neusch C, Schnierle S, Moser A. Selegiline induces dopamine as predictors of time to onset of response to fluoxetine in major release through ATP-sensitive potassium channels in the rat depressive disorder. J Clin Psychiatry 65(4):543–546, 2004. caudate-putamen in vitro. Neurochem Int 31(2):307–311, 1997. 87. March JS, Kobak KA, Jefferson JW, et al. A double-blind, placebo- 63. Tuomisto J, Smith DF. Effects of tranylcypromine enantiomers controlled trial of fluvoxamine versus imipramine in outpatients on monoamine uptake and release and imipramine binding. with major depression. J Clin Psychiatry 51(5):200–202, 1990. J Neural Transm 65(2):135–145, 1986. 88. Feighner JP, Boyer WF, Meredith CH, et al. A placebo-controlled 64. Dingemanse J, Wallnofer A, Gieschke R, et al. Pharmacoki­ inpatient comparison of fluvoxamine maleate and imipramine netic and pharmacodynamic interactions between fluoxetine in major depression. Int Clin Psychopharmacol 4(3):239–244, and moclobemide in the investigation of development of 1989. the “serotonin syndrome. Clin Pharmacol Ther 63(4):403–413, 89. Masand PS, Gupta S. Selective serotonin-reuptake inhibitors: an 1998. update. Harv Rev Psychiatry 7(2):69–84, 1999.

Downloaded for Rohul Amin ([email protected]) at Uniformed Services Univ of the Health Sciences from ClinicalKey.com by Elsevier on September 21, 2018. For personal use only. No other uses without permission. Copyright ©2018. Elsevier Inc. All rights reserved. Antidepressants 505.e3

90. Fava M, Rankin M. Sexual functioning and SSRIs. J Clin Psychiatry 114. Nonacs R, Cohen LS. Assessment and treatment of depression 63(Suppl. 5):13–16, discussion 23–25, 2002. during pregnancy: an update. Psychiatr Clin North Am 26(3):547– 43 91. Rothschild AJ. Selective serotonin reuptake inhibitor-induced 562, 2003. sexual dysfunction: efficacy of a drug holiday. Am J Psychiatry 115. Rosenbaum JF, Fava M, Hoog SL, et al. Selective serotonin 152(10):1514–1516, 1995. reuptake inhibitor discontinuation syndrome: a randomized 92. Fava M, Graves LM, Benazzi F, et al. A cross-sectional study of . Biol Psychiatry 44(2):77–87, 1998. the prevalence of cognitive and physical symptoms during long- 116. Fava M. Prospective studies of adverse events related to antide- term antidepressant treatment. J Clin Psychiatry 67(11):1754– pressant discontinuation. J Clin Psychiatry 67(Suppl. 4):14–21, 1759, 2006. 2006. 93. Gerber PE, Lynd LD. Selective serotonin-reuptake inhibitor- 117. Michelson D, Fava M, Amsterdam J, et al. Interruption of induced movement disorders. Ann Pharmacother 32(6):692–698, selective serotonin reuptake inhibitor treatment. Double- 1998. blind, placebo-controlled trial. Br J Psychiatry 176:363–368, 94. Gill HS, DeVane CL, Risch SC. Extrapyramidal symptoms associ- 2000. ated with cyclic antidepressant treatment: a review of the litera- 118. Rosenbaum JF, Zajecka J. Clinical management of antidepressant ture and consolidating hypotheses. J Clin Psychopharmacol discontinuation. J Clin Psychiatry 58(Suppl. 7):37–40, 1997. 17(5):377–389, 1997. 119. Hemeryck A, Belpaire FM. Selective serotonin reuptake inhibi- 95. Steur EN. Increase of Parkinson disability after fluoxetine medi- tors and cytochrome P-450 mediated drug-drug interactions: an cation. Neurology 43(1):211–213, 1993. update. Curr Drug Metab 3(1):13–37, 2002. 96. Leo RJ, Lichter DG, Hershey LA. Parkinsonism associated with 120. Pastuszak A, Schick-Boschetto B, Zuber C, et al. Pregnancy fluoxetine and cimetidine: a case report. J Geriatr Psychiatry outcome following first-trimester exposure to fluoxetine Neurol 8(4):231–233, 1995. (Prozac). JAMA 269(17):2246–2248, 1993. 97. Leo RJ. Movement disorders associated with the serotonin selec- 121. Sanchez C, Hyttel J. Comparison of the effects of antidepressants tive reuptake inhibitors. J Clin Psychiatry 57(10):449–454, 1996. and their metabolites on reuptake of biogenic amines and on 98. Van de Vijver DA, Roos RA, Jansen PA, et al. Start of a selective receptor binding. Cell Mol Neurobiol 19(4):467–489, 1999. serotonin reuptake inhibitor (SSRI) and increase of antiparkin- 122. Harvey AT, Rudolph RL, Preskorn SH. Evidence of the dual sonian drug treatment in patients on levodopa. Br J Clin Phar- mechanisms of action of venlafaxine. Arch Gen Psychiatry macol 54(2):168–170, 2002. 57(5):503–509, 2000. 99. Lake MB, Birmaher B, Wassick S, et al. Bleeding and selective 123. Roseboom PH, Kalin NH. Neuropharmacology of venlafaxine. serotonin reuptake inhibitors in childhood and adolescence. Depress Anxiety 12(Suppl. 1):20–29, 2000. J Child Adolesc Psychopharmacol 10(1):35–38, 2000. 124. Wellington K, Perry CM. Venlafaxine extended-release: a review 100. Alderman CP, Seshadri P, Ben-Tovim DI. Effects of serotonin of its use in the management of major depression. CNS Drugs reuptake inhibitors on hemostasis. Ann Pharmacother 15(8):643–669, 2001. 30(11):1232–1234, 1996. 125. Thase ME, Entsuah AR, Rudolph RL. Remission rates during 101. Sussman N, Ginsberg DL, Bikoff J. Effects of nefazodone on treatment with venlafaxine or selective serotonin reuptake inhib- body weight: a pooled analysis of selective serotonin reuptake itors. Br J Psychiatry 178:234, 2001. inhibitor- and imipramine-controlled trials. J Clin Psychiatry 126. Rudolph RL. Achieving remission from depression with venla- 62(4):256–260, 2001. faxine and venlafaxine extended release: a literature review of 102. Michelson D, Amsterdam JD, Quitkin FM, et al. Changes in comparative studies with selective serotonin reuptake inhibitors. weight during a 1-year trial of fluoxetine. Am J Psychiatry Acta Psychiatr Scand Suppl (415):24–30, 2002. 156(8):1170–1176, 1999. 127. Smith D, Dempster C, Glanville J, et al. Efficacy and tolerability 103. Fava M, Judge R, Hoog SL, et al. Fluoxetine versus sertraline and of venlafaxine compared with selective serotonin reuptake paroxetine in major depressive disorder: changes in weight with inhibitors and other antidepressants: a meta-analysis. Br J Psy- long-term treatment. J Clin Psychiatry 61(11):863–867, 2000. chiatry 180:396–404, 2002. 104. Papakostas GI, Petersen T, Montoya H, et al. Treatment-related 128. Saiz-Ruiz J, Ibanez A, Diaz-Marsa M, et al. Efficacy of venlafaxine adverse events and outcome in a clinical trial of fluoxetine for in major depression resistant to selective serotonin reuptake depression. Ann Clin Psychiatry 15(3–4):187–192, 2003. inhibitors. Prog Neuropsychopharmacol Biol Psychiatry 26(6):1129– 105. Calabrese JR, Londborg PD, Shelton MD, et al. Citalopram treat- 1134, 2002. ment of fluoxetine-intolerant depressed patients. J Clin Psychiatry 129. Mbaya P. Safety and efficacy of high dose of venlafaxine XL in 64(5):562–567, 2003. treatment resistant major depression. Hum Psychopharmacol 106. Thase ME, Ferguson JM, Lydiard RB, et al. Citalopram treatment 17(7):335–339, 2002. of paroxetine-intolerant depressed patients. Depress Anxiety 130. Kaplan EM. Efficacy of venlafaxine in patients with major depres- 16(3):128–133, 2002. sive disorder who have unsustained or no response to selective 107. Thase ME, Blomgren SL, Birkett MA, et al. Fluoxetine treatment serotonin reuptake inhibitors: an open-label, uncontrolled of patients with major depressive disorder who failed initial study. Clin Ther 24(7):1194–1200, 2002. treatment with sertraline. J Clin Psychiatry 58(1):16–21, 1997. 131. Schweitzer I, Burrows G, Tuckwell V, et al. Sustained response to 108. Brown WA, Harrison W. Are patients who are intolerant to one open-label venlafaxine in drug-resistant major depression. J Clin serotonin selective reuptake inhibitor intolerant to another? Psychopharmacol 21(2):185–189, 2001. J Clin Psychiatry 56(1):30–34, 1995. 132. Mitchell PB, Schweitzer I, Burrows G, et al. Efficacy of venlafax- 109. Golden RN, Nemeroff CB, McSorley P, et al. Efficacy and toler- ine and predictors of response in a prospective open-label study ability of controlled-release and immediate-release paroxetine in of patients with treatment-resistant major depression. J Clin Psy- the treatment of depression. J Clin Psychiatry 63(7):577–584, chopharmacol 20(4):483–487, 2000. 2002. 133. Poirier MF, Boyer P. Venlafaxine and paroxetine in treatment- 110. Misra M, Papakostas GI, Klibanski A. Effects of psychiatric dis- resistant depression. Double-blind, randomised comparison. orders and psychotropic medications on prolactin and bone Br J Psychiatry 175:12–16, 1999. metabolism. J Clin Psychiatry 65(12):1607–1618, 2004. 134. de Montigny C, Silverstone PH, Debonnel G, et al. Venlafaxine 111. Papakostas GI, Miller KK, Petersen T, et al. Serum prolactin levels in treatment-resistant major depression: a Canadian multicenter, among outpatients with major depressive disorder during the open-label trial. J Clin Psychopharmacol 19(5):401–406, 1999. acute phase of treatment with fluoxetine. J Clin Psychiatry 135. Nierenberg AA, Feighner JP, Rudolph R, et al. Venlafaxine for 67(6):952–957, 2006. treatment-resistant unipolar depression. J Clin Psychopharmacol 112. Richards JB, Papaioannou A, Adachi JD, et al. Canadian Multi- 14(6):419–423, 1994. centre Osteoporosis Study Research Group. Effect of selective 136. Kornbluh R, Papakostas GI, Petersen T, et al. A survey of pre- serotonin reuptake inhibitors on the risk of fracture. Arch Intern scribing preferences in the treatment of refractory depression: Med 167(2):188–194, 2007. recent trends. Psychopharmacol Bull 35(3):150–156, 2001. 113. Henry JA, Alexander CA, Sener EK. Relative mortality from over- 137. Nelson JC. Safety and tolerability of the new antidepressants. dose of antidepressants. BMJ 310(6974):221–224, 1995. J Clin Psychiatry 58(Suppl. 6):26–31, 1997.

Downloaded for Rohul Amin ([email protected]) at Uniformed Services Univ of the Health Sciences from ClinicalKey.com by Elsevier on September 21, 2018. For personal use only. No other uses without permission. Copyright ©2018. Elsevier Inc. All rights reserved. 505.e4 PART X Treatment Approaches

138. Jaffee MS, Bostwick JM. Buspirone as an antidote to venlafaxine- in elderly patients with major depressive episode. Acta Psychiatr induced bruxism. Psychosomatics 41(6):535–536, 2000. Scand 97(2):157–165, 1998. 139. Brown ES, Hong SC. Antidepressant-induced bruxism success- 160. Steen A, Den Boer JA. A double-blind six months comparative fully treated with . J Am Dent Assoc 130(10):1467– study of milnacipran and clomipramine in major depressive 1469, 1999. disorder. Int Clin Psychopharmacol 12(5):269–281, 1997. 140. Clayton AH, Pradko JF, Croft HA, et al. Prevalence of sexual 161. von Frenckell R, Ansseau M, Serre C, et al. Pooling two control- dysfunction among newer antidepressants. J Clin Psychiatry led comparisons of milnacipran (F2207) and amitriptyline in 63(4):357–366, 2002. endogenous inpatients. A new approach in dose ranging studies. 141. Montejo AL, Llorca G, Izquierdo JA, et al. Incidence of sexual Int Clin Psychopharmacol 5(1):49–56, 1990. dysfunction associated with antidepressant agents: a prospective 162. Ansseau M, von Frenckell R, Mertens C, et al. Controlled com- multicenter study of 1022 outpatients. Spanish Working Group parison of two doses of milnacipran (F 2207) and amitriptyline for the Study of Psychotropic-Related Sexual Dysfunction. J Clin in major depressive inpatients. Psychopharmacology (Berl) Psychiatry 62(Suppl. 3):10–21, 2001. 98(2):163–168, 1998. 142. Entsuah R, Chitra R. A benefit-risk analysis of once-daily venla- 163. Macher JP, Sichel JP, Serre C, et al. Double-blind placebo- faxine extended release (XR) and venlafaxine immediate release controlled study of milnacipran in hospitalized patients with (IR) in outpatients with major depression. Psychopharmacol Bull major depressive disorders. Neuropsychobiology 22(2):77–82, 33(4):671–676, 1997. 1989. 143. Rudolph RL, Derivan AT. The safety and tolerability of venlafax- 164. Lecrubier Y, Pletan Y, Solles A, et al. Clinical efficacy of milnacip- ine hydrochloride: analysis of the clinical trials database. J Clin ran: placebo-controlled trials. Int Clin Psychopharmacol 11(Suppl. Psychopharmacol 16(3 Suppl. 2):54S–59S, 1996. 4):29–33, 1996. 144. Thase ME. Effects of venlafaxine on blood pressure: a meta- 165. Sambunaris A, Bose A, Gommoll CP, et al. A phase III, double- analysis of original data from 3744 depressed patients. J Clin blind, placebo-controlled, flexible-dose study of levomilnacip- Psychiatry 59(10):502–508, 1998. ran extended-release in patients with major depressive disorder. 145. Fava M, Mulroy R, Alpert J, et al. Emergence of adverse events J Clin Psychopharmacol 34(1):47–56, 2014. following discontinuation of treatment with extended-release 166. Montgomery SA, Mansuy L, Ruth AC, et al. The efficacy of venlafaxine. Am J Psychiatry 154(12):1760–1762, 1997. extended-release levomilnacipran in moderate to severe major 146. Gonzalez-Pinto A, Gutierrez M, Gonzalez N, et al. Efficacy and depressive disorder: secondary and post-hoc analyses from a safety of venlafaxine-ECT combination in treatment-resistant randomized, double-blind, placebo-controlled study. Int Clin depression. J Neuropsychiatry Clin Neurosci 14(2):206–209, 2002. Psychopharmacol 29(1):26–35, 2014. 147. Bymaster FP, Dreshfield-Ahmad LJ, Threlkeld PG, et al. Com- 167. Bakish D, Bose A, Gommoll C, et al. Levomilnacipran ER 40 mg parative affinity of duloxetine and venlafaxine for serotonin and and 80 mg in patients with major depressive disorder: a phase norepinephrine transporters in vitro and in vivo, human serot- III, randomized, double-blind, fixed-dose, placebo-controlled onin receptor subtypes, and other neuronal receptors. Neuropsy- study. J Psychiatry Neurosci 39(1):40–49, 2014. chopharmacology 25(6):871–880, 2001. 168. Wong EH, Sonders MS, Amara SG, et al. Reboxetine: a pharma- 148. Thase ME, Lu Y, Joliat MJ, et al. Remission in placebo-controlled cologically potent, selective, and specific norepinephrine trials of duloxetine with an SSRI comparator. Presented at the reuptake inhibitor. Biol Psychiatry 47(9):818–829, 2000. Annual Meeting of the American Psychiatric Association, 2005. 169. Montgomery S, Ferguson JM, Schwartz GE. The antidepressant 149. Perahia DG, Kajdasz DK, Desaiah D, et al. Symptoms following efficacy of reboxetine in patients with severe depression. J Clin abrupt discontinuation of duloxetine treatment in patients with Psychopharmacol 23(1):45–50, 2003. major depressive disorder. J Affect Disord 89(1–3):207–212, 170. Andreoli V, Caillard V, Deo RS, et al. Reboxetine, a new noradren- 2005. aline selective antidepressant, is at least as effective as fluoxetine 150. Detke MJ, Lu Y, Goldstein DJ, et al. Duloxetine 60 mg once daily in the treatment of depression. J Clin Psychopharmacol 22(4):393– dosing versus placebo in the acute treatment of major depres- 399, 2002. sion. J Psychiatr Res 36(6):383–390, 2002. 171. Versiani M, Amin M, Chouinard G. Double-blind, placebo- 151. Detke MJ, Lu Y, Goldstein DJ, et al. Duloxetine, 60 mg once controlled study with reboxetine in inpatients with severe daily, for major depressive disorder: a randomized double-blind major depressive disorder. J Clin Psychopharmacol 20(1):28–34, placebo-controlled trial. J Clin Psychiatry 63(4):308–315, 2002. 2000. 152. Goldstein DJ, Mallinckrodt C, Lu Y, et al. Duloxetine in the 172. Massana J. Reboxetine versus fluoxetine: an overview of efficacy treatment of major depressive disorder: a double-blind clinical and tolerability. J Clin Psychiatry 59(Suppl. 14):8–10, 1998. trial. J Clin Psychiatry 63(3):225–231, 2002. 173. Clayton AH, Zajecka J, Ferguson JM, et al. Lack of sexual dysfunc- 153. Mochizuki D, Tsujita R, Yamada S, et al. Neurochemical and tion with the selective noradrenaline reuptake inhibitor reboxet- behavioural characterization of milnacipran, a serotonin and ine during treatment for major depressive disorder. Int Clin noradrenaline reuptake inhibitor in rats. Psychopharmacology Psychopharmacol 18(3):151–156, 2003. (Berl) 162(3):323–332, 2002. 174. Preti A. Tomoxetine (Eli Lilly & Co). Curr Opin Investig Drugs 154. Clerc G, Milnacipran/Fluvoxamine Study Group. Antidepressant 3(2):272–277, 2002. efficacy and tolerability of milnacipran, a dual serotonin and 175. Chouinard G, Annable L, Bradwejn J. An early phase II clinical noradrenaline reuptake inhibitor: a comparison with fluvox­ trial of tomoxetine (LY139603) in the treatment of newly admit- amine. Int Clin Psychopharmacol 16(3):145–151, 2001. ted depressed patients. Psychopharmacology (Berl) 83(1):126– 155. Guelfi JD, Ansseau M, Corruble E, et al. A double-blind com- 128, 1984. parison of the efficacy and safety of milnacipran and fluoxetine 176. Wernicke JF, Kratochvil CJ. Safety profile of atomoxetine in the in depressed inpatients. Int Clin Psychopharmacol 13(3):121–128, treatment of children and adolescents with ADHD. J Clin Psy- 1998. chiatry 63(Suppl. 12):50–55, 2002. 156. Ansseau M, Papart P, Troisfontaines B, et al. Controlled compari- 177. Thase ME, Haight BR, Richard N, et al. Remission rates following son of milnacipran and fluoxetine in major depression. Psychop- antidepressant therapy with bupropion or selective serotonin harmacology (Berl) 114(1):131–137, 1994. reuptake inhibitors: a meta-analysis of original data from 7 ran- 157. Papakostas GI, Fava M. A meta-analysis of clinical trials compar- domized controlled trials. J Clin Psychiatry 66(8):974–981, 2005. ing milnacipran, a serotonin-norepinephrine reuptake inhibitor, 178. Papakostas GI. Dopaminergic-based pharmacotherapies for with a selective serotonin reuptake inhibitor for the treatment depression. Eur Neuropsychopharmacol 16(6):391–402, 2006. of major depressive disorder. Eur Neuropsychopharmacol 17(1): 179. Trivedi MH, Rush AJ, Carmody TJ, et al. Do bupropion SR and 32–36, 2007. sertraline differ in their effects on anxiety in depressed patients? 158. van Amerongen AP, Ferrey G, Tournoux A. A randomised, J Clin Psychiatry 62(10):776–781, 2001. double-blind comparison of milnacipran and imipramine in the 180. Papakostas GI, Nutt DJ, Hallett LA, et al. Resolution of sleepi- treatment of depression. J Affect Disord 72(1):21–31, 2002. ness and fatigue in major depressive disorder: a comparison of 159. Tignol J, Pujol-Domenech J, Chartres JP, et al. Double-blind bupropion and the selective serotonin reuptake inhibitors. Biol study of the efficacy and safety of milnacipran and imipramine Psychiatry 60(12):1350–1355, 2006.

Downloaded for Rohul Amin ([email protected]) at Uniformed Services Univ of the Health Sciences from ClinicalKey.com by Elsevier on September 21, 2018. For personal use only. No other uses without permission. Copyright ©2018. Elsevier Inc. All rights reserved. Antidepressants 505.e5

181. Thase ME, Clayton AH, Haight BR, et al. A double-blind com- 204. Ferguson JM, Shrivastava RK, Stahl SM, et al. Reemergence of parison between bupropion XL and venlafaxine XR: sexual func- sexual dysfunction in patients with major depressive disorder: 43 tioning, antidepressant efficacy, and tolerability. J Clin double-blind comparison of nefazodone and sertraline. J Clin Psychopharmacol 26(5):482–488, 2006. Psychiatry 62(1):24–29, 2001. 182. Coleman CC, King BR, Bolden-Watson C, et al. A placebo- 205. Toofanny N, Maddens ME. Reversible penile priapism associated controlled comparison of the effects on sexual functioning of with nefazodone. J Am Geriatr Soc 50(9):1610–1611, 2002. bupropion sustained release and fluoxetine. Clin Ther 206. Brodie-Meijer CC, Diemont WL, Buijs PJ. Nefazodone-induced 23(7):1040–1058, 2001. clitoral priapism. Int Clin Psychopharmacol 14(4):257–258, 1999. 183. Coleman CC, Cunningham LA, Foster VJ, et al. Sexual dysfunc- 207. Carvajal GP, Garcia D, Sanchez SA, et al. Hepatotoxicity associ- tion associated with the treatment of depression: a placebo- ated with the new antidepressants. J Clin Psychiatry 63(2):135– controlled comparison of bupropion sustained release and 137, 2002. sertraline treatment. Ann Clin Psychiatry 11(4):205–215, 1999. 208. Stewart DE. Hepatic adverse reactions associated with nefazo- 184. Segraves RT, Kavoussi R, Hughes AR, et al. Evaluation of sexual done. Can J Psychiatry 47(4):375–377, 2002. functioning in depressed outpatients: a double-blind compari- 209. Ehrentraut S, Rothenhausler HB, Gerbes AL, et al. Acute liver son of sustained-release bupropion and sertraline treatment. failure in nefazodone therapy? A case report. Nervenarzt J Clin Psychopharmacol 20(2):122–128, 2000. 73(7):686–689, 2002. 185. Croft H, Settle E Jr, Houser T, et al. A placebo-controlled com- 210. Khan A, Sambunaris A, Edwards J, et al. Vilazodone in the treat- parison of the antidepressant efficacy and effects on sexual func- ment of major depressive disorder: efficacy across symptoms and tioning of sustained-release bupropion and sertraline. Clin Ther severity of depression. Int Clin Psychopharmacol 29(2):86–92, 21(4):643–658, 1999. 2013. 186. Kavoussi RJ, Segraves RT, Hughes AR, et al. Double-blind com- 211. Clayton AH, Kennedy SH, Edwards JB, et al. The effect of vila- parison of bupropion sustained release and sertraline in zodone on sexual function during the treatment of major depressed outpatients. J Clin Psychiatry 58(12):532–537, 1997. depressive disorder. J Sex Med 10(10):2465–2476, 2013. 187. Modell JG, Katholi CR, Modell JD, et al. Comparative sexual side 212. Citrome L. Vilazodone for major depressive disorder: a system- effects of bupropion, fluoxetine, paroxetine, and sertraline. Clin atic review of the efficacy and safety profile for this newly Pharmacol Ther 61(4):476–487, 1997. approved antidepressant—what is the number needed to treat, 188. Feighner JP, Gardner EA, Johnston JA, et al. Double-blind com- number needed to harm and likelihood to be helped or harmed? parison of bupropion and fluoxetine in depressed outpatients. Int J Clin Pract 66(4):356–368, 2012. J Clin Psychiatry 52(8):329–335, 1991. 213. Mahableshwarkar AR, Jacobsen PL, Chen Y, et al. A randomised, 189. Settle EC, Stahl SM, Batey SR, et al. Safety profile of sustained- double-blind, placebo-controlled, duloxetine-referenced study release bupropion in depression: results of three clinical trials. of the efficacy and tolerability of vortioxetine in the acute treat- Clin Ther 21(3):454–463, 1999. ment of adults with generalised anxiety disorder. Int J Clin Pract 190. Weihs KL, Settle EC Jr, Batey SR, et al. Bupropion sustained 68(1):49–59, 2014. release versus paroxetine for the treatment of depression in the 214. Alam MY, Jacobsen PL, Chen Y, et al. Safety, tolerability, and elderly. J Clin Psychiatry 61(3):196–202, 2000. efficacy of vortioxetine (Lu AA21004) in major depressive disor- 191. Weihs KL, Houser TL, Batey SR, et al. Continuation phase treat- der: results of an open-label, flexible-dose, 52-week extension ment with bupropion SR effectively decreases the risk for relapse study. Int Clin Psychopharmacol 29(1):36–44, 2014. of depression. Biol Psychiatry 51(9):753–761, 2002. 215. Bersani G, Pozzi F, Marini S, et al. 5-HT2 receptor antagonism 192. Dunner DL, Zisook S, Billow AA, et al. A prospective safety sur- in dysthymic disorder: a double-blind placebo-controlled study veillance study for bupropion sustained-release in the treatment with ritanserin. Acta Psychiatr Scand 83(4):244–248, 1991. of depression. J Clin Psychiatry 59(7):366–373, 1998. 216. Nappi G, Sandrini G, Granella F, et al. A new 5-HT2 antagonist 193. Papakostas GI, Fava M. A meta-analysis of clinical trials compar- (ritanserin) in the treatment of chronic headache with depres- ing the serotonin (5HT)-2 receptor antagonists trazodone and sion. A double-blind study vs amitriptyline. Headache 30(7):439– nefazodone with a selective serotonin reuptake inhibitor for the 444, 1990. treatment of major depressive disorder. Eur Psychiatry 22(7):444– 217. Loo H, Hale A, D’haenen H. Determination of the dose of ago- 447, 2007. melatine, a melatoninergic agonist and selective 5-HT(2C) 194. Nierenberg AA, Adler LA, Peselow E, et al. Trazodone for antagonist, in the treatment of major depressive disorder: a antidepressant-associated insomnia. Am J Psychiatry 151(7):1069– placebo-controlled dose range study. Int Clin Psychopharmacol 1072, 1994. 17(5):239–247, 2002. 195. Nierenberg AA, Cole JO, Glass L. Possible trazodone potentia- 218. Kennedy SH, Emsley R. Placebo-controlled trial of agomelatine tion of fluoxetine: a case series. J Clin Psychiatry 53(3):83–85, in the treatment of major depressive disorder. Eur Neuropsychop- 1992. harmacol 16(2):93–100, 2006. 196. Thompson JW Jr, Ware MR, Blashfield RK. Psychotropic medica- 219. Montgomery SA, Olie JP, Ahokas A, et al. Antidepressant efficacy tion and priapism: a comprehensive review. J Clin Psychiatry of agomelatine in MDD: a meta-analysis of three pivotal trials. Pre- 51(10):430–433, 1990. sented at the Annual Meeting of the European College of Neu- 197. Bardin ED, Krieger JN. Pharmacological priapism: comparison ropsychopharmacology, 2006. of trazodone- and papaverine-associated cases. Int Urol Nephrol 220. Rickels K, Amsterdam J, Clary C, et al. Buspirone in depressed 22(2):147–152, 2002. outpatients: a controlled study. Psychopharmacol Bull 26(2):163– 198. Saenz de Tejada I, Ware JC, Blanco R, et al. Pathophysiology of 167, 1990. prolonged penile erection associated with trazodone use. J Urol 221. Rickels K, Amsterdam JD, Clary C, et al. Buspirone in major 145(1):60–64, 1991. depression: a controlled study. J Clin Psychiatry 52(1):34–38, 199. Fernandes NF, Martin RR, Schenker S. Trazodone-induced hepa- 1991. totoxicity: a case report with comments on drug-induced hepa- 222. Robinson DS, Rickels K, Feighner J, et al. Clinical effects of the totoxicity. Am J Gastroenterol 95(2):532–535, 2000. 5-HT1A partial agonists in depression: a composite analysis of 200. de Meester A, Carbutti G, Gabriel L, et al. Fatal overdose with buspirone in the treatment of depression. J Clin Psychopharmacol trazodone: case report and literature review. Acta Clin Belg 10(3 Suppl.):67S–76S, 1990. 56(4):258–261, 2001. 223. Fabre LF. Buspirone in the management of major depression: a 201. Pisani F, Spina E, Oteri G. Antidepressant drugs and seizure placebo-controlled comparison. J Clin Psychiatry 51(Suppl.):55– susceptibility: from in vitro data to clinical practice. Epilepsia 61, 1990. 40(Suppl. 10):S48–S56, 1999. 224. Jenkins SW, Robinson DS, Fabre LF Jr, et al. Gepirone in the 202. Cyr M, Brown CS. Nefazodone: its place among antidepressants. treatment of major depression. J Clin Psychopharmacol 10(3 Ann Pharmacother 30(9):1006–1012, 1996. Suppl.):77S–85S, 1990. 203. Sargent PA, Sharpley AL, Williams C, et al. 5-HT2C receptor 225. McGrath PJ, Stewart JW, Quitkin FM, et al. Gepirone treatment activation decreases appetite and body weight in obese subjects. of atypical depression: preliminary evidence of serotonergic Psychopharmacology (Berl) 133(3):309–312, 1997. involvement. J Clin Psychopharmacol 14(5):347–352, 1994.

Downloaded for Rohul Amin ([email protected]) at Uniformed Services Univ of the Health Sciences from ClinicalKey.com by Elsevier on September 21, 2018. For personal use only. No other uses without permission. Copyright ©2018. Elsevier Inc. All rights reserved. 505.e6 PART X Treatment Approaches

226. Wilcox CS, Ferguson JM, Dale JL, et al. A double-blind trial of 246. Pancrazio JJ, Kamatchi GL, Roscoe AK, et al. Inhibition of neu- low- and high-dose ranges of gepirone-ER compared with ronal Na+ channels by antidepressant drugs. J Pharmacol Exp Ther placebo in the treatment of depressed outpatients. Psychophar- 284(1):208–214, 1998. macol Bull 32(3):335–342, 1996. 247. Harrigan RA, Brady WJ. ECG abnormalities in tricyclic antide- 227. Feiger AD. A double-blind comparison of gepirone extended pressant ingestion. Am J Emerg Med 17(4):387–393, 1999. release, imipramine, and placebo in the treatment of outpatient 248. Wedin GP, Oderda GM, Klein-Schwartz W, et al. Relative toxicity major depression. Psychopharmacol Bull 32(4):659–665, 1996. of cyclic antidepressants. Ann Emerg Med 15(7):797–804, 1986. 228. Feiger AD, Heiser JF, Shrivastava RK, et al. Gepirone extended- 249. Danish University Antidepressant Group. Citalopram: clinical release: new evidence for efficacy in the treatment of major effect profile in comparison with clomipramine. A controlled depressive disorder. J Clin Psychiatry 64(3):243–249, 2003. multicenter study. Psychopharmacology (Berl) 90(1):131–138, 229. Newton RE, Marunycz JD, Alderdice MT, et al. Review of the 1986. side-effect profile of buspirone. Am J Med 80(3B):17–21, 1986. 250. Danish University Antidepressant Group. Paroxetine: a selective 230. Papakostas GI, Homberger CH, Fava M. A meta-analysis of clini- serotonin reuptake inhibitor showing better tolerance, but cal trials comparing mirtazapine with a selective serotonin weaker antidepressant effect than clomipramine in a controlled reuptake inhibitor for the treatment of major depressive disor- multicenter study. J Affect Disord 18(4):289–299, 1990. der. J Psychopharmacol 22(8):843–848, 2008. 251. Roose SP, Glassman AH, Attia E, et al. Comparative efficacy of 231. Benkert O, Szegedi A, Philipp M, et al. Mirtazapine orally disin- selective serotonin reuptake inhibitors and tricyclics in the tegrating tablets versus venlafaxine extended release: a double- treatment of melancholia. Am J Psychiatry 151(12):1735–1739, blind, randomized multicenter trial comparing the onset of 1994. antidepressant response in patients with major depressive disor- 252. Robinson RG, Schultz SK, Castillo C, et al. Nortriptyline versus der. J Clin Psychopharmacol 26(1):75–78, 2006. fluoxetine in the treatment of depression and in short-term 232. Guelfi JD, Ansseau M, Timmerman L, et al. Mirtazapine- recovery after stroke: a placebo-controlled, double-blind study. Venlafaxine Study Group: mirtazapine versus venlafaxine in hos- Am J Psychiatry 157(3):351–359, 2000. pitalized severely depressed patients with melancholic features. 253. Navarro V, Gasto C, Torres X, et al. Citalopram versus nortriptyl- J Clin Psychopharmacol 21(4):425–431, 2001. ine in late-life depression: a 12-week randomized single-blind 233. Hong CJ, Hu WH, Chen CC, et al. A double-blind, randomized, study. Acta Psychiatr Scand 103(6):435–440, 2001. group-comparative study of the tolerability and efficacy of 6 254. Sindrup SH, Jensen TS. Efficacy of pharmacological treatments weeks’ treatment with mirtazapine or fluoxetine in depressed of neuropathic pain: an update and effect related to mechanism Chinese patients. J Clin Psychiatry 64(8):921–926, 2003. of drug action. Pain 83(3):389–400, 1999. 234. Leinonen E, Skarstein J, Behnke K, et al. Efficacy and tolerability 255. O’Malley PG, Jackson JL, Santoro J, et al. Antidepressant therapy of mirtazapine versus citalopram: a double-blind, randomized for unexplained symptoms and symptom syndromes. J Fam Pract study in patients with major depressive disorder. Nordic Antide- 48(12):980–990, 1999. pressant Study Group. Int Clin Psychopharmacol 14(6):329–337, 256. Anderson IM, Tomenson BM. Treatment discontinuation with 1999. selective serotonin reuptake inhibitors compared with tricyclic 235. Benkert O, Szegedi A, Kohnen R. Mirtazapine compared with antidepressants: a meta-analysis. BMJ 310(6992):1433–1438, paroxetine in major depression. J Clin Psychiatry 61(9):656–663, 1995. 2000. 257. Huang CC. Persistent tardive dyskinesia associated with amoxa- 236. Wheatley DP, van Moffaert M, Timmerman L, et al. Mirtazapine: pine therapy. Am J Psychiatry 143(8):1069–1070, 1986. efficacy and tolerability in comparison with fluoxetine in 258. Hayashi Y, Ohyagi Y, Inoue I, et al. A case of amoxapine-induced patients with moderate to severe major depressive disorder. tardive dystonia successfully treated with a low dose anti- Mirtazapine-Fluoxetine Study Group. J Clin Psychiatry 59(6):306– cholinergic agent. Rinsho Shinkeigaku 40(4):367–371, 2000. 312, 1998. 259. Okun MS, Watts RL. Depression associated with Parkinson’s 237. Schatzberg AF, Kremer C, Rodrigues HE, et al. Mirtazapine vs. disease: clinical features and treatment. Neurology 58(4 Suppl. Paroxetine Study Group: Double-blind, randomized compari- 1):S63–S70, 2002. son of mirtazapine and paroxetine in elderly depressed patients. 260. Fava M. Weight gain and antidepressants. J Clin Psychiatry Am J Geriatr Psychiatry 10(5):541–550, 2002. 61(Suppl. 11):37–41, 2000. 238. Wade A, Crawford GM, Angus M, et al. A randomized, double- 261. Goodnick PJ. Use of antidepressants in treatment of comorbid blind, 24-week study comparing the efficacy and tolerability of diabetes mellitus and depression as well as in diabetic neuropa- mirtazapine and paroxetine in depressed patients in primary thy. Ann Clin Psychiatry 13(1):31–41, 2001. care. Int Clin Psychopharmacol 18(3):133–141, 2003. 262. Pollock BG, Perel JM, Paradis CF, et al. Metabolic and physio- 239. Behnke K, Sogaard J, Martin S, et al. Mirtazapine orally disinte- logic consequences of nortriptyline treatment in the elderly. Psy- grating tablet versus sertraline: a prospective onset of action chopharmacol Bull 30(2):145–150, 1994. study. J Clin Psychopharmacol 23(4):358–364, 2003. 263. Peeters FP, deVries MW, Vissink A. Risks for oral health with the 240. Gelenberg AJ, McGahuey C, Laukes C, et al. Mirtazapine substi- use of antidepressants. Gen Hosp Psychiatry 20(3):150–154, tution in SSRI-induced sexual dysfunction. J Clin Psychiatry 1998. 61(5):356–360, 2000. 264. Roose SP, Glassman AH, Giardina EG, et al. Tricyclic antidepres- 241. Moller HJ, Riehl T, Dietzfelbinger T, et al. A controlled study of sants in depressed patients with cardiac conduction disease. Arch the efficacy and safety of mianserin and maprotiline in outpa- Gen Psychiatry 44(3):273–275, 1987. tients with major depression. Int Clin Psychopharmacol 6(3):179– 265. Thayssen P, Bjerre M, Kragh-Sorensen P, et al. Cardiovascular 192, 1991. effect of imipramine and nortriptyline in elderly patients. Psy- 242. Moller HJ, Kasper S, Muller H, et al. A controlled study of the chopharmacology (Berl) 74(4):360–364, 1981. efficacy and safety of mianserin and amitriptyline in depressive 266. Roberts RL, Joyce PR, Mulder RT, et al. A common P-glycoprotein inpatients. Pharmacopsychiatry 28(6):249–252, 1995. polymorphism is associated with nortriptyline-induced postural 243. Wilcox CS, Cohn JB, Katz BB, et al. A double-blind, placebo- hypotension in patients treated for major depression. Pharma- controlled study comparing mianserin and amitriptyline in cogenomics J 2(3):191–196, 2002. moderately depressed outpatients. Int Clin Psychopharmacol 267. Ray WA, Griffin MR, Malcolm E. Cyclic antidepressants and the 9(4):271–279, 1994. risk of hip fracture. Arch Intern Med 151(4):754–756, 1991. 244. Licht RW, Qvitzau S. Treatment strategies in patients with major 268. Ramaekers JG. Antidepressants and driver impairment: empirical depression not responding to first-line sertraline treatment. A evidence from a standard on-the-road test. J Clin Psychiatry randomised study of extended duration of treatment, dose 64(1):20–29, 2003. increase or mianserin augmentation. Psychopharmacology (Berl) 269. Nelson JC, Kennedy JS, Pollock BG, et al. Treatment of major 161(2):143–151, 2002. depression with nortriptyline and paroxetine in patients with 245. Anderson IM. Selective serotonin reuptake inhibitors versus tri- ischemic heart disease. Am J Psychiatry 156(7):1024–1028, 1999. cyclic antidepressants: a meta-analysis of efficacy and tolerabil- 270. Pisani F, Oteri G, Costa C, et al. Effects of psychotropic drugs on ity. J Affect Disord 58(1):19–36, 2000. seizure threshold. Drug Saf 25(2):91–110, 2002.

Downloaded for Rohul Amin ([email protected]) at Uniformed Services Univ of the Health Sciences from ClinicalKey.com by Elsevier on September 21, 2018. For personal use only. No other uses without permission. Copyright ©2018. Elsevier Inc. All rights reserved. Antidepressants 505.e7

271. Henry JA, Alexander CA, Sener EK. Relative mortality from over- 287. White PD, Cleary KJ. An open study of the efficacy and adverse dose of antidepressants. BMJ 310(6974):221–224, 1995. effects of moclobemide in patients with the chronic fatigue syn- 43 272. Perry PJ, Zeilmann C, Arndt S. Tricyclic antidepressant concen- drome. Int Clin Psychopharmacol 12(1):47–52, 1997. trations in plasma: an estimate of their sensitivity and specificity 288. Natelson BH, Cheu J, Pareja J, et al. Randomized, double blind, as a predictor of response. J Clin Psychopharmacol 14(4):230–240, controlled placebo-phase in trial of low dose phenelzine in the 1994. chronic fatigue syndrome. Psychopharmacology (Berl) 124(3):226– 273. Flint AJ, Rifat SL. The effect of sequential antidepressant treat- 230, 1996. ment on geriatric depression. J Affect Disord 36(3–4):95–105, 289. Hickie IB, Wilson AJ, Wright JM, et al. A randomized, double- 1996. blind placebo-controlled trial of moclobemide in patients with 274. McGrath PJ, Stewart JW, Nunes EV, et al. A double-blind cross­ chronic fatigue syndrome. J Clin Psychiatry 61(9):643–648, over trial of imipramine and phenelzine for outpatients with 2000. treatment-refractory depression. Am J Psychiatry 150(1):118–123, 290. Hannonen P, Malminiemi K, Yli-Kerttula U, et al. A randomized, 1993. double-blind, placebo-controlled study of moclobemide and 275. Nolen WA, Haffmans PM, Bouvy PF, et al. Monoamine oxidase amitriptyline in the treatment of fibromyalgia in females without inhibitors in resistant major depression. A double-blind com- psychiatric disorder. Br J Rheumatol 37:1279–1286, 1998. parison of brofaromine and tranylcypromine in patients resist- 291. Natelson BH, Cheu J, Hill N, et al. Single-blind, placebo phase-in ant to tricyclic antidepressants. J Affect Disord 28(3):189–197, trial of two escalating doses of selegiline in the chronic fatigue 1993. syndrome. Neuropsychobiology 37(3):150–154, 1998. 276. Thase ME, Frank E, Mallinger AG, et al. Treatment of imipramine- 292. Goldenberg D, Mayskiy M, Mossey C, et al. A randomized, resistant recurrent depression, III. Efficacy of monoamine double-blind, crossover trial of fluoxetine and amitriptyline in oxidase inhibitors. J Clin Psychiatry 53(1):5–11, 1992. the treatment of fibromyalgia. Arthitis Rheum 39:1852–1859, 277. Nolen WA. Tranylcypromine in depression resistant to cyclic 1996. antidepressions. Prog Neuropsychopharmacol Biol Psychiatry 13(1– 293. Wolfe F, Cathey MA, Hawley DJ. A double-blind placebo control- 2):155–158, 1989. led trial of fluoxetine in fibromyalgia. Scand J Rheumatol 23:255– 278. Nolen WA, van de Putte JJ, Dijken WA, et al. Treatment strategy 259, 1994. in depression. II. MAO inhibitors in depression resistant to cyclic 294. Cantini F, Bellandi F, Niccoli L, et al. Fluoxetine associated with antidepressants: two controlled crossover studies with tranylcy- in the treatment of primary fibromyalgia. promine versus L-5-hydroxytryptophan and . Acta Minerva Med 85:97–100, 1994. Psychiatr Scand 78(6):676–683, 1988. 295. Norregaard J, Volkmann H, Danneskiold-Samsoe B. A ran­ 279. McGrath PJ, Stewart JW, Harrison W, et al. Treatment of tricyclic domized controlled trial of citalopram in the treatment of fibro- refractory depression with a monoamine oxidase inhibitor anti- myalgia. Pain 61:445–449, 1995. depressant. Psychopharmacol Bull 23(1):169–172, 1987. 296. Vercoulen JH, Swanink CM, Zitman FG, et al. Randomized 280. Stabl M, Kasas A, Blajev B, et al. A double-blind comparison of double-blind placebo-controlled study of fluoxetine in chronic moclobemide and thioridazine versus moclobemide and fatigue syndrome. Lancet 347(9005):858–861, 1996. placebo in the treatment of refractory, severe depression. J Clin 297. Bodkin JA, Amsterdam JD. Transdermal selegiline in major Psychopharmacol 15(4 Suppl. 2):41S–45S, 1995. depression: a double-blind, placebo-controlled, parallel-group 281. Hoencamp E, Haffmans PM, Dijken WA, et al. Brofaromine study in outpatients. Am J Psychiatry 159(11):1869–1875, 2002. versus addition to maprotiline. A double-blind study in 298. Amsterdam JD. A double-blind, placebo-controlled trial of the maprotiline refractory depressed outpatients. J Affect Disord safety and efficacy of selegiline transdermal system without 30(3):219–227, 1994. dietary restrictions in patients with major depressive disorder. 282. Sunderland T, Cohen RM, Molchan S, et al. High-dose selegiline J Clin Psychiatry 64(2):208–214, 2003. in treatment-resistant older depressive patients. Arch Gen Psychia- 299. Richard IH, Kurlan R, Tanner C, et al. Serotonin syndrome and try 51(8):607–615, 1994. the combined use of deprenyl and an antidepressant in Parkin- 283. Volz HP, Faltus F, Magyar I, et al. Brofaromine in treatment- son’s disease. Parkinson Study Group. Neurology 48(4):1070– resistant depressed patients—a comparative trial versus tranylcy- 1077, 1997. promine. J Affect Disord 30(3):209–217, 1994. 300. Philipp M, Tiller JW, Baier D, et al. Comparison of moclobemide 284. Georgotas A, Friedman E, McCarthy M, et al. Resistant geriatric with selective serotonin reuptake inhibitors (SSRIs) on sexual depressions and therapeutic response to monoamine oxidase function in depressed adults. The Australian and German Study inhibitors. Biol Psychiatry 18(2):195–205, 1983. Groups. Eur Neuropsychopharmacol 10(5):305–314, 2000. 285. Amsterdam JD, Berwish NJ. High dose tranylcypromine therapy 301. Harrison W, Stewart J, Lovelace R, et al. Case report of carpal for refractory depression. Pharmacopsychiatry 22(1):21–25, 1989. tunnel syndrome associated with tranylcypromine. Am J Psychia- 286. Thase ME, Trivedi MH, Rush AJ. MAOIs in the contemporary try 140(9):1229–1230, 1983. treatment of depression. Neuropsychopharmacology 12(3):185– 219, 1995.

Downloaded for Rohul Amin ([email protected]) at Uniformed Services Univ of the Health Sciences from ClinicalKey.com by Elsevier on September 21, 2018. For personal use only. No other uses without permission. Copyright ©2018. Elsevier Inc. All rights reserved. 505.e8 PART X Treatment Approaches

MULTIPLE CHOICE QUESTIONS ○ Desipramine Select the appropriate answer. ○ Doxepin ○ Imipramine Q1 Which of the following agents was noted in the 1950s to possess antidepressant efficacy in patients suffering Q7 Which of the following agents is LEAST likely to cause from tuberculosis? anticholinergic symptoms? ○ ○ Amitriptyline Diphenhydramine ○ ○ Doxepin Iproniazid ○ ○ Paroxetine Nortriptyline ○ ○ Phenelzine Phenelzine ○ ○ Thioridazine Q2 Which of the following syndromes is characterized by alterations in cognition (e.g., disorientation and MULTIPLE CHOICE ANSWERS confusion), behavior (e.g., agitation and restlessness), Q1 The answer is: Iproniazid. autonomic nervous system function (e.g., fever, shivering, diaphoresis, and diarrhea), and The precursors of two of the major contemporary antidepres- neuromuscular activity (e.g., ataxia, hyperreflexia, and sant families, the monoamine oxidase inhibitors (MAOIs) and myoclonus)? the tricyclic antidepressants (TCAs), were discovered by seren- dipity in the 1950s. ○ Catatonia Specifically, the administration of iproniazid, an antimycobac- ○ Delirium terial agent, was first noted to possess antidepressant effects in ○ Neuroleptic malignant syndrome depressed patients suffering from tuberculosis. Shortly there- after, iproniazid was found to inhibit MAO, and was involved ○ Serotonin syndrome in the catabolism of serotonin, norepinephrine, and dopamine. ○ Wernicke’s encephalopathy In parallel, imipramine was initially developed as an antihis- tamine, but Kuhn and colleagues in 1958 discovered that of Q3 Which of the following is the time that most experts some 500 imipramine-treated patients with various psychiat- favor for a continuation of antidepressant therapy ric disorders, only those with endogenous depression with following the achievement of remission after an mental and motor retardation showed a remarkable improve- episode of major depression? ment during 1 to 6 weeks of daily imipramine therapy. The ○ 2 weeks same compound was also found to inhibit the reuptake of serotonin and norepinephrine. Thus, it was the discovery of ○ 4 weeks the antidepressant effects of iproniazid and imipramine that ○ 8 weeks led to the development of the MAOIs and TCAs, but also such discovery was instrumental in the formulation of the monoam- ○ 16 weeks ine theory of depression. In turn, guided by this theory, the ○ 26 weeks subsequent development of compounds selective for the reuptake of either serotonin or norepinephrine or both was Q4 Which of the following SSRIs is not FDA-approved for designed, rather than accidental. As a result, over the last few the treatment of depression? decades, chemical alterations of these first antidepressants have resulted in the creation of a wide variety of monoamine- ○ Citalopram based antidepressants with a variety of mechanisms of action. ○ Fluvoxamine Q2 The answer is: Serotonin syndrome. ○ Fluoxetine Monoamine oxidase inhibitors (MAOIs) act by inhibiting ○ Paroxetine MAO, an enzyme found on the outer membrane of mitochon- ○ Sertraline dria, where it catabolizes a number of monoamines including dopamine, norepinephrine, and serotonin. After reuptake, Q5 Which of the following drugs is BEST classified as a norepinephrine, serotonin, and dopamine are either reloaded norepinephrine dopamine reuptake inhibitor? into vesicles for subsequent release or broken down by the enzyme MAO. ○ Atomoxetine MAO is present in two forms (MAOA and MAOB), which ○ Bupropion differ in their substrate preferences, inhibitor specificities, ○ Duloxetine tissue expression, and cell distribution. MAOA preferentially oxidizes serotonin and is irreversibly inactivated by low con- ○ Trazodone centrations of the acetylenic inhibitor clorgyline. MAOB pref- ○ Venlafaxine erentially oxidizes phenylethylamine (PEA) and benzylamine and is irreversibly inactivated by low concentrations of par- Q6 Which of the following is classified as a secondary gyline and deprenyl. Dopamine, tyramine, and tryptamine amine tricyclic antidepressant? are substrates for both forms of MAO. In the gastrointestinal (GI) tract and the liver, MAO catabolizes a number of dietary ○ Amitriptyline pressor amines (such as dopamine, tyramine, tryptamine, and ○ Clomipramine phenylethylamine).

Downloaded for Rohul Amin ([email protected]) at Uniformed Services Univ of the Health Sciences from ClinicalKey.com by Elsevier on September 21, 2018. For personal use only. No other uses without permission. Copyright ©2018. Elsevier Inc. All rights reserved. Antidepressants 505.e9

For this reason, consumption of certain foods (that contain only for the treatment of obsessive-compulsive disorder high levels of dietary amines) while on an MAOI may precipi- (OCD). 43 tate a hypertensive crisis, characterized by hypertension, hyper- Due to their relatively low side-effect burden, the starting dose pyrexia, tachycardia, tremulousness, and cardiac arrhythmias. of SSRIs is often the minimally effective daily dose: 10 mg for The same reaction may also occur during co-administration of escitalopram (Lexapro); 20 mg for fluoxetine (Prozac), parox- dopaminergic agents and MAOIs, while the co-administration etine (Paxil), and citalopram (Celexa); 50 mg for sertraline of MAOIs with other antidepressants that potentiate serotonin (Zoloft); and 100 mg for fluvoxamine (Luvox). Starting at could result in serotonin syndrome due to toxic CNS serotonin lower doses, and increasing the dose shortly thereafter (i.e., levels. The serotonin syndrome is characterized by alterations after 1 to 2 weeks) may further improve tolerability. Maximum in cognition (e.g., disorientation and confusion), behavior therapeutic doses for SSRIs are typically one-fold to two-fold (e.g., agitation and restlessness), autonomic nervous system greater than the starting dose. function (e.g., fever, shivering, diaphoresis, and diarrhea), and neuromuscular activity (e.g., ataxia, hyperreflexia, and myo- Q5 The answer is: Bupropion. clonus). Since MAO enzymatic activity requires approximately 14 days to be restored, such food or medications should be The norepinephrine dopamine reuptake inhibitor (NDRI) avoided for 2 weeks after the discontinuation of an irreversible bupropion appears to be as effective as the SSRIs in the treat- MAOI (“MAOI washout period”). Serotonergic and dopamin- ment of depressive as well as anxiety symptoms in depression. ergic antidepressants are typically discontinued 2 weeks before Bupropion is a phenethylamine compound that is effective for the initiation of an MAOI, with the exception of fluoxetine, the treatment of major depressive disorder (MDD). Bupro- which needs to be discontinued 5 weeks in advance due to its pion is structurally related to amphetamine and the sym- relatively longer half-life. pathomimetic diethylpropion and it primarily blocks the reuptake of dopamine and norepinephrine and has minimal Q3 The answer is: 26 weeks. or no affinity for postsynaptic receptors. Although some Originally, based on studies with tricyclic antidepressants researchers have argued that the NDRI bupropion’s effect on (TCAs), patients with unipolar depressive disorders were norepinephrine is primarily through an increase in presynap- observed to be at high risk for relapse when treatment was tic release, there is still convincing evidence for binding of discontinued within the first 16 weeks of therapy. Therefore, both norepinephrine and dopamine transporters. in treatment-responders, most experts favor a continuation of Venlafaxine, duloxetine, and milnacipran share the property antidepressant therapy for a minimum of 6 months following of being relatively potent reuptake inhibitors of serotonin and the achievement of remission. The value of continuation norepinephrine and are therefore considered selective nore- therapy for several months to prevent relapse into the original pinephrine reuptake inhibitors (SNRIs). Of these, only venla- episode has also been established for virtually all of the newer faxine and duloxetine are approved for the treatment of agents. depression in the United States. Risk of recurrence after this 6- to 8-month continuation period Reboxetine acts by selectively inhibiting the norepinephrine (that is, the development of a new episode after recovery from transporter, thereby increasing synaptic norepinephrine levels. the index episode) is particularly elevated in patients with a Atomoxetine also selectively inhibits the reuptake of nore- chronic course before recovery, residual symptoms, and mul- pinephrine. These agents are called norepinephrine reuptake tiple prior episodes (three or more). For these individuals, the inhibitors (NRIs). optimal duration of maintenance treatment is unknown but it is measured in years. Based on research to date, prophylactic Trazodone and nefazodone are classified as serotonin receptor efficacy of an antidepressant has been observed for as long as antagonists/agonists, while mirtazapine and mianserin are 5 years with clear benefit. alpha2-adrenergic receptor antagonists. In contrast to the initial expectation that maintenance therapy Q6 The answer is: Desipramine. would be effective at dosages lower than that required for The TCAs may be subdivided into tertiary amines and second- acute treatment, the current consensus is that full-dose therapy ary amines (their demethylated secondary amine derivatives). is required for effective prophylaxis. About 20% to 30% of In addition, maprotiline (Ludiomil), which is classified as a patients who are treated with each of the classes of antidepres- tetracyclic antidepressant, is commonly grouped with the sants will experience a return of depressive symptoms despite TCAs, due to similarities in dosing, mechanism of action, and continued treatment. In such patients, a dose increase of the side effects. antidepressant is typically the first-line approach. Tertiary amine TCAs include amitriptyline (Elavil, Adepril), Q4 The answer is: Fluvoxamine. imipramine (Tofranil, Antidepril), trimipramine (Surmontil, The overall efficacy of the SSRIs in the treatment of depression Herphonal), clomipramine (Anafranil, Clopress), and doxepin is equivalent to the other antidepressants, while all six of the (Sinequan, Deptran). SSRIs appear to be equally effective in the treatment of depres- Secondary amine TCAs are nortriptyline (Pamelor, Aventyl), sion. Due to their favorable side-effect profile, the SSRIs are desipramine (Norpramin, Metylyl), protriptyline (Vivactil, used as first-line treatment in the overwhelming majority of Concordin), and amoxapine (Ascendin, Defanyl). cases, with more than 90% of clinicians indicating that SSRIs were their first line of treatment. Despite the tolerability and Q7 The answer is: Phenelzine the widespread efficacy of the SSRIs, there is mounting evi- Anticholinergic-like side effects occur in MAOI-treated dence to suggest that depressed patients with certain charac- patients, although they are not due to muscarinic antagonism. teristics (including co-morbid anxiety disorders and a greater These side effects are less severe than those seen with TCAs, number of somatic symptoms [such as pain, headaches, and although patients on phenelzine may experience dry mouth. fatigue]) respond less well to SSRIs than those without such Elderly patients may develop constipation or urinary reten- characteristics. tion. Alternatively, nausea and diarrhea have been reported by Only one of the SSRIs, fluvoxamine, is not approved for the some patients. Sweating, flushing, or chills also may occur treatment of depression in the United States, as it is approved with their use.

Downloaded for Rohul Amin ([email protected]) at Uniformed Services Univ of the Health Sciences from ClinicalKey.com by Elsevier on September 21, 2018. For personal use only. No other uses without permission. Copyright ©2018. Elsevier Inc. All rights reserved.